LCCW Adjunctive Care 3 - Cumulative Final (with practice questions)

Pataasin ang iyong marka sa homework at exams ngayon gamit ang Quizwiz!

What type of exercises place the lowest strains on the PCL? A. Open Kinetic Chain in Extension B. Open Kinetic Chain in Flexion C. Closed Kinetic Chain in Extension D. Closed Kinetic Chain in Flexion

*A. Open Kinetic Chain in Extension*

Which of the following is a clinical finding of acetabular labral tears: A. Painful clicking B. Non-painful locking C. Restricted movement in the hip

*A. Painful clicking* Also -painful locking -giving way in the hip

Which of the following is/are appropriate ortho tests for TFC injuries: A. TFCC grind test B. Piano key test C. Fovea sign D. Extension/Flexion AROM and PROM

*A. TFCC grind test* *B. Piano key test* *C. Fovea sign*

The lateral meniscus is attached to: A. ACL B. PCL C. MCL D. LCL

*B*. PCL And Popliteus

The minimum normal q-angle measurement is: A. 5º B. 10º C. 15º D. 25º

*B. 10º*

External snapping (extra-articular) is often the result of: A. Labral tears B. Iliotibial band rubs over the greater trochanter C. Iliopsoas tendon moves over a bony prominence D. Iliotibial band rubs over the lesser trochanter

*B. Iliotibial band rubs over the greater trochanter*

Which of the following repetitive movements is false with regards acetabular labral tears: A. Pivoting B. Twitching C. Twisting D. Cutting Maneuvers

*B. Twitching* = FALSE A, C and D* = TRUE

What test is crucial for evaluating an injury to the MCL? A. Valgus stress test with knee in 20 degrees of flexion B. Valgus stress test with knee in 30 degrees of flexion C. Valgus stress test with knee in 20 degrees of extension D. Valgus stress test with knee in 30 degrees of extension

*B. Valgus stress test with knee in 30 degrees of flexion*

Which one is (are) the non-surgical treatments for an UCL rupture? A. Immobilization of thumb for 2 weeks B. AROM and PROM should occur at 3-4 weeks C. Grip strength activities should start around week 4 D. Bracing at 1 month when return to play is advisable

*B.* A. Immobilization of thumb for *4* weeks C. Grip strength activities should start around week *6* D. Bracing at *2* month when return to play is advisable

What should you DDX PFPS with?

*Meniscus issues.* -Patients often complain of the patella "giving way" during straight-ahead activities or stair climbing (versus instability owing to ACL or PCL injury, which is typically associated with giving way during pivoting or changing directions. *Patellar subluxation.* -Subluxation typically lacks a history of trauma found with ACL-related instability.

________, aka anterior knee pain, is one of the most common lower extremity conditions reported in physically active populations, affecting ____ in 4 people.

*Patellofemoral pain syndrome (PFPS)*, aka anterior knee pain, is one of the most common lower extremity conditions reported in physically active populations, affecting *1* in 4 people.

T or F: A large effusion is suggestive of an intra-articular injury?

*True*

T or F: If the tibia medially rotates this allows my knees to come together and increase the Q-angle

*True*

T or F: Simple meniscal tears are more likely to heal than complex meniscal tears.

*True*

T or F: The medial collateral ligament attaches to the medial meniscus

*True*

T or F: The ACL rehabilitation protocol may include having the patient balance on foam pads.

*True* Neuromuscular and proprioceptive retraining may include perturbation exercises = exercises that disturb the balance, alignment and stability = to make it stronger

T or F: Anterior knee pain is one of the most common LE conditions

*True* anterior knee pain = patellofemoral pain syndrome

What are the suspected contributors to PFPS?

-Imbalance of the vastus medialis and vastus lateralis. -Abnormally large Q angle. -Tibiofemoral abduction angular impulse. -High pelvis width to femoral length ratio.

What functional factors contribute to chronic ankle instability?

-Impaired proprioception/JPS -Impaired neuromuscular control -Impaired postural control -Strength deficits

What is the purpose of the acetabular labrum?

-Important for proprioception. -Distributes synovial fluid within the joint (so when injured this does not happen and there is resultant arthritc changes).

What are some possible signs and symptoms of PFPS?

-Instability -Patellar subluxation -Overuse or training errors. -Localization of pain. -Crepitance -Aggravating activities -Swelling -Weakness -Night pain

What are the possible extra-articular hip injuries?

-Internal coxa saltans -External coxa saltans -Gluteal tears -Muscle strains -Piriformis syndrome -Slipped capital femoral epiphysis -Fractures

What are the three categories of hip injuries?

-Intra-articular -Extra-articular -Hip mimickers (DDXs)

The ____ is the most commonly injured ligament of the ankle, followed by the ________.

1) Anterior talofibular ligament (ATFL). 2) Calcaneofibular ligament (CFL).

MOI for TFC is most commonly due to:

1. FOOSH 2. Rotational injuries or axial load injuries

Name 5 possible causes for younger patients with cuff symptoms:

1. Glenohumeral instability 2. Tears of the superior labrum 3. Scapulothoracic dyskinesia 4. Core stability deficits 5. Poor biomechanics

What are the 3 categories of hip injuries?

1. Intra-articular 2. Extra-articular 3. Hip mimickers

What are the 2 types of femoroacetabular impingement?

1. Pincer impingement 2. Cam impingement

What are the phases of treatment for and MCL elbow injury?

1. Reduce pain and inflammation, promote healing, avoid loss of ROM. Brace with acute injuries and not with chronic. Avoid valgus stress to elbow. 2. Restore full ROM, strength, and entire flexibility of the entire upper extremity. 3. Add internal and external rotation exercises incorporated into the program later to avoid excessive valgus stress to the elbow.

____ ounce of fluid in the knee (joint effusion) will cause arthrogenic muscle inhibition.

1/3

What is the minimum normal Q-angle measurement?

10 degrees

A hinged elbow brace should be increased ________ degrees per week.

10-15

What age group has the highest risk for ankle sprains?

10-19 year olds

What percentage of all elbow injuries are dislocations?

11-28%

Elbow dislocation accounts for _______% of all elbow injuries.

11-28

Structural injury to rotator cuff is common in patients over _______. If the patient is over ______, assume rotator cuff tear unless proven otherwise.

40 60

Components of Hx important in establishing the correct dx include...

Age Presenting complaint (pain, instability, weakness, crepitus & stiffness) Onset of sx Duration of sx Response to previous trx General health

M/c direction of GH instability

Anterior Inferior

Isometric exercises include ___________ and ______________.

Balance Proprioceptive activation

Co-morbidities associated with Frozen Shoulder

Diabetes Mellitus Hypothyroidism (bilateral)

Trauma leading to detachment of superior glenoid labrum

FOOSH Long hx of participation in overhead sports

MOI for elbow dislocation

FOOSH w/ arm abducted MVA Direct trauma Sport injury or high energy mechanism

Progression of frozen shoulder

Freezing Frozen Thaw

Patient's hx post-traumatic elbow stiffness

Hx of trauma, previous surgery, septic arthritis

MCL of Forearm includes...

Medial epicondyle Anterior, Posterior, Transverse Ligament @ ulna Annular ligament (over radius head)

The MCL of the elbow is the ______________ to valgus load.

Primary restraint

Normal function of the shoulder requires the coordinated movements of the...

SC joint AC joint GH joint Scapulothoracic articulation Motion interface between rotator cuff & overlying coracoacromial arch

Night pain & _________ position (AKA zero position) are due to primary rotator cuff pathology.

Saha

With elbow dislocation there is a loss of ________________ most common & _____% have contractures.

Terminal extension 60

The scapula is mobile on the _______________.

Thoracic wall

Ligaments and capsules of the shoulder limit ____________ and ____________ of the humeral head on the glenoid.

Translation Rotation

To establish the correct dx, ___________ is key.

History

Biceps Tendonitis patient demographic

Older age Rotator cuff disorder in hx

Static stabilizers of the shoulder

Superior GH ligament Middle GH ligament Inferior GH ligament

Principles of Rehabilitation

1. Pain control & regain coordinated motion 2. Strengthen & re-educate 3. mm re-education through "learned motor patterns" 4. Patterns position shoulder complex in "pre-determined" ways & activate mm in precise synchronization

A patient with MCL/ulnar nerve injury will have a history of _______________ with throwing during late cocking phase & early acceleration or ____________ due to pain.

Chronic elbow pain Inability to throw

Crepitus is common in ______________________ or thickening of the cuff during ________________ & scarring of subacromial space.

Chronic full-thickness cuff tears Chronic tendinosis

Scapulothoracic dyskinesia is secondary to _________________.

Irritation of the rotator cuff

Initial treatment/rehab of MCL injury of the elbow is to decrease ____________ & ____________, increase _____________, & avoid decreased ROM & __________.

Pain Inflammation Soft tissue healing Valgus stress

Sx of biceps Tendonitis

Pain @ bicep referred to anterior shoulder (increases with forearm rotation - door knob, wine pour)

Detachment of Superior Glenoid Labrum (AKA _________________)

SLAP lesion

Higher levels of exercise are only appropriate with adequate _____________ & __________________.

Strength base Full ROM

Structural injury to the rotator cuff is most common with ___________ or _________.

Trauma Acute overuse

Post-Traumatic Elbow stiffness extrinsic causes

1. Skin contractures 2. Capsular & collateral ligament contracture 3. Heterotropic ossification 4. Injury @ brachialis or triceps (scarring, fibrosis, limit motion)

Scapulothoracic dyskinesia has an ___________________ onset with participation of a new _______________ or ___________.

Atraumatic insidious Recreational activity Occupational task

When taking hx of suspected GH instability, it is important to note ...

Degree of instability Onset Direction of instability

Anterior/Inferior laxity pattern =

External rotation w/ or w/o abduction

Comorbidities present with Post-Traumatic Elbow stiffness

Hemophilia >> causing hemoarthrosis, or spastic neuropathy >> neuropathic joint degeneration

Superior GH ligament is an ___________ stabilizer.

Inferior

Crepitus with ROM indicates __________, __________ or _____________.

Loose body Fracture Degenerative changes

Non-surgical treatment of Post-traumatic surgical disorder

NSAIDs Ice/heat Massage Iontophoresis US Electrical stimulation Static, progressive, adjustable splints (for F/E deficits) Supinator/Pronator splints (pts w/ sx <6mo-1 year w/ decreased articular Involvement)

After active ROM of the shoulder, ____________ should follow. These exercises include ______________ and _______________.

Passive ROM Codman's circumdxn Wand exercises (or pully system)

S/sx for structural injury to rotator cuff

1. Pain local to subacromial area or anterior/lateral corner of acromion with radiation down lateral arm to delt tubercle 2. Dull ache 3. Internal rotation causes sharper stabbing pain 4. Radiation of pain does not go past elbow

Assessing the ROM of the shoulder should include...

Apleys 1 Reach across to opposite shoulder Apleys 2

Muscle strengthening in the rehab protocol depends on the ___________.

Diagnosis

Terrible triad =

Elbow dislocation Radial head fracture Coronoid process fracture

Secondary function of the shoulder is a ____________ which forces from the larger, stronger muscles of the ________ & ________, are passed to the muscles of the arm, forearm & hand.

Funnel Legs Trunk (Generating power in throwing motions or serving motions)

Young patient with rotator cuff symptoms is most likely only ___________, rather than a structural injury. Pathology and sx are frequently secondary to _____________.

Irritation (tendinosis) Occult primary pathology

_________________ are important early in the rehab protocol & should begin side lying with isometrics & isotonic or closed chain.

Scapular stabilizers

Passive ROM should begin with the patient _____________ and as pain decreases move them to _________________.

Supine Seated/standing

Arthroscopic test of elbow joint would show greater than ________ of space between the __________ & __________.

1mm Coronoid Medial humerus

If there is a difference of ______ degrees or greater with internal rotation, a _______________ is indicated.

20 Shoulder pathology

Acute traumatic MCL injury can sometimes be __________, while chronic throwing injuries will not ____________.

Braced (tape) Be braced

Pain of the rotator cuff is decreased with arm below __________ and raised between ____________ active elevation & abduction.

Breast level 90-120 degrees

Closed kinetic chain exercises include a ______________ of agonist & antagonist, stabilizing the ___________.

Co-contraction GH joint

__________________ is the most important factor in determining the success or failure of rehabilitation.

Establishing the correct dx

Posterior instability is suggested when there is pain with __________.

Follow through (forward flexion, adduction, internal rotation)

Hypertrophic osteophytes may __________ & form loose bodies which would further _____________.

Fracture Limit extension

Adhesive capsulitis (AKA ________________)

Frozen shoulder

Occult primary pathology causing rotator cuff symptoms in young patients includes...

GH instability Tear of superior labrum Scapulothroacic dyskinesia Core instability Poor biomechanics

_______ intensity & short duration is ____________ for limited elbow ROM, and may produce ________________.

High Contraindicated Hypertrophic ossification

Clinical presentation of MCL/Ulnar nerve injury

Hx: Repetitive f/e with valgus stress Hypertrophic flexor/pronator mass (often w/ ulnar nerve paresthesia) 4-5 digit PTN posteromedial elbow pain

Examples of pain control

Ice Rest Stabilization Elevation Stim-TENS Natural supplements Decrease inflammation

Pain associated with primary inferior distraction force is _____________ laxity of the capsule & ______________ instability.

Inferior Multidirectional

Calcified Tendonitis is associated with ____________, ____________ & _____________ onset of lateral shoulder pain, and is m/c in ___________ (age) people.

Insidious Rapid Severe Middle-aged

Functional progressions of elbow rehab include _________ & _________ exercises later to avoid excessive _____________.

Internal External rotation Valgus stress

_________ is the method of imaging that will show MCL damage.

MRI

Repetitive overloading of the MCL causes ____________ & _____________ of the ligament & may cause _____________ & possible ______________, which increases if the joint is unstable.

Microtears Inflammation Failure Ulnar nerve injury

To restore extension with a flexion contracture patient, it would be important to ______________ the Joint & give ______-load, ________ duration stretching.

Mobilize Low Long

Initial treatment of flexion contracture includes ___________ & __________, dynamic splinting in the night when sleeping, joint _________ & ROM exercises at end ranges several times a day.

Moist heat US Mobilization

Frozen shoulder hx

No trauma although pt may remember minor shoulder injury

First step of shoulder rehab is _____________.

Normalize ROM (active first)

The most functional of all open-chain exercises are _________________.

Plyometric exercises (stretch-shortening cycle of mm-tendon unit)

D2 F/E exercises has the patient moving their arm through ____________________, which enhances stabilization of the GH joint through AROM.

Pre-determined patterns

Reactive osteophytes may occur @ the ____________ & corresponding ______________ with flexion contracture causing ______________, which may impinge & limit ____________.

Proximal olecranon Olecranon fossa Kissing osteophytes Terminal extension

Pain relief should be achieved by ___________ & ___________.

Rest Avoidance (immobilize, cryotherapy, ultrasound, galvanic stimulation, oral/injectable)

Stability of the shoulder depends on the _______________ that cross it.

Soft tissue structures

Clinical presentation for elbow dislocation

Soft tissue swelling Obvious deformities Terrible triad

MCL/Ulnar nerve injury is common in ___________________ due to the large ___________________.

Throwing athletes (pitchers, quarterbacks, javelin throwers) Valgus torque (angular velocity F to Extension)

Elbow dislocation is the most common dislocation in kids _______________ & 2nd most common in adults, second to ___________.

Under 10 y.o. Shoulder

Inferior GH ligament is the most important _____________ with the shoulder in 90 degrees of _________ & __________, which is the most unstable position of the shoulder.

Anterior stabilizer Abduction External rotation

Middle GH ligament provides stability against _________________ with arm in ____________ & ___________ under 90 degrees.

Anterior translation External rotation Abduction

Subjective sign of GH instability

Apprehension with arm in specific positions

____-____% of individuals have chronic ankle instability after an acute ________ ankle sprain.

*10-30%* of individuals have chronic ankle instability after an acute *lateral* ankle sprain.

____% of people with ankle sprain sustained a second sprain within ____ months. Low-grade acute ankle sprains (grades I or II) resulted in a ________ risk of reinjury than high-grade (grade III) sprains.

*18%* of people with ankle sprain sustained a second sprain within *24* months. Low-grade acute ankle sprains (grades I or II) resulted in a *higher* risk of reinjury than high-grade (grade III) sprains. -This is because with a grade I or II sprain, there is very little rehab associated with recovery. With Grade III, there is extensive rehab and therefore more awareness and a lower change for reinjury.

Which of the following statement(s) are true of the knee joint capsule: A. Provides joint stability B. Moves synovial fluid in the joint c. Is not connected to the bursae and hence provides flexibility to the joint D. Contain Pacinian mechnoreceptors that are sensitive to pressure and deformation

*A* = true B. Moves synovial fluid *around* the joint C. *Is* connected to the bursae and hence provides flexibility to the joint D. Contain *Ruffini* mechanoreceptors that are sensitive to pressure and deformation

Which of these two tests can determine ACL translation? A. Anterior Drawer Test B. Lachman Test C. Apley Test D. Clarke's Test

*A* and *B*

What two malalignment problems should be investigated in patients with patellofemoral pain? A. Compensatory pronation with medial tibial rotation B. Compensatory pronation with lateral tibial rotation C. Hip lateral rotation with lateral tibial rotation D. Hip medial rotation with medial tibial rotation

*A* and *D*

The medial meniscus is attached to: A. ACL B. PCL C. MCL D. LCL

*A*, *C* And Semimembranosus tendon

Which of the following should you perform after inflammation is gone from external coxa saltans? A. Gentle stretching B. Hip Adduction strengthening exercises C. Hip internal rotation strengthening exercises D. Hip abduction strengthening exercises E. Hip external rotation strengthening exercises

*A, D and E*

Which of the following are crucial elements for history: A. Age B. Weight C. Old complaints D. Duration of symptoms E. BP and RR

*A, D* •age •presenting complaint •onset of symptoms •duration of symptoms •response to previous treatment •general health

Which of the following is used to test the *integrity* of the anterior cruciate ligament? A. Anterior Drawer Test B. Lachman Test C. Apley Test D. Clarke's Test

*A. Anterior Drawer Test* B. Lachman Test = used to diagnose ACL injury C. Apley Test = evaluate problems of the meniscus D. Clarke's Test = test for patellofemoral syndrome

Which of the following will show anterior translation: A. Anterior Drawer Test B. Lachman Test C. Apley Test D. Clarke's Test

*A. Anterior Drawer Test* And *B. Lachman Test*

Persistent AMI can: A. Decrease in knee stability B. Increase in knee stability C. Increase of function or performance D. Decreases injury

*A. Decrease in knee stability* and if AMI does persist: Decrease of function or performance Increase injury

What do labral lesions lead to: A. Early osteoarthritic changes B. Knee is constant flexion C. Instability D. Foot flare

*A. Early osteoarthritic changes* *C. Instability*

Which statement is true: A. Excessive rearfoot pronation influences the patella's alignment because it increases tibial medial rotation B. Excessive rearfoot pronation influences the patella's alignment because it increases tibial lateral rotation C. Excessive forefoot pronation influences the patellas alignment because it increases tibial medial rotation D. Excessive forefoot pronation influences the patellas alignment because it increase tibial lateral rotation

*A. Excessive rearfoot pronation influences the patella's alignment because it increases tibial medial rotation* and changes the quadriceps tendon pull on the patella

Is when full passive motion is present but active terminal knee extension does not occur - what does this describe? A. Extensor lag B. Flexor lag C. Extensor log D. Flexor log

*A. Extensor Lag*

Intra-articular snapping hip is often the result of: A. Labral tears B. Iliotibial band rubs over the greater trochanter C. Iliopsoas tendon moves over a bony prominence D. Iliotibial band rubs over the lesser trochanter

*A. Labral tears*

What statement is true of arthrogenic muscle inhibition A. May be observed after knee injury, knee surgery or patients with knee joint arthritis B. Is a protective reflex inhibition that protects the knee from a strong hamstring contraction C. AMI allows the quad to fire properly D. AMI causes a quick rehabilitation E. AMI is not associated with joint effusion

*A. May be observed after knee injury, knee surgery or patients with knee joint arthritis* B. Is a protective reflex inhibition that protects the knee from a strong *quad* contraction C. AMI *prevents* the quad to fire properly D. AMI causes a *delay in* rehabilitation E. AMI *is associated* with joint effusion

What type of exercise place the lowest strains on the PCL but results in elevated patello-femoral joint forces? A. Open Kinetic Chain in Extension B. Open Kinetic Chain in Flexion C. Closed Kinetic Chain in Extension D. Closed Kinetic Chain in Flexion

*A. Open Kinetic Chain in Extension*

When during the gait cycle is the femur weight supported by the posterior menisci? Anterior menisci?

-Going into flexion: posterior minisci. -Goint into extension: anterior menisci.

Which of the following passage is correct for PCL rehabilitation? A. With open chain activities there is a tremendous force exerted on the PCL during extension exercises B. Passive motion can be safely performed through the entire range of flexion and extension. C. Active closed kinetic chain activities of any kind, in passive ROM, should be used cautiously when rehabilitating the PCL D. Closed kinetic chain flexion exercises generate extremely high forces in the PCL and should be avoided altogether.

*B.* Passive motion can be safely performed through the entire range of flexion and extension. A. With open chain activities there is a tremendous force exerted on the PCL during *flexion* exercises C. Active closed kinetic chain activities of any kind, *in any ROM*, should be used cautiously when rehabilitating the PCL D. *Open* kinetic chain flexion exercises generate extremely high forces in the PCL and should be avoided altogether.

With regards to ACL - which statement is true? A. Provides posterior stability B. Provides anterior stability C. Provides rotational stability D. ACL tears are least common than PCL tears

*B.* and *C* D. ACL tears are *more* common than PCL tears

________ deficits have been identified in most patients with chronic ankle instability, so 4 weeks of ________ training significantly improves self-reported function, static postural control, and dynamic postural control.

*Balance* deficits have been identified in most patients with chronic ankle instability, so 4 weeks of *balance* training significantly improves self-reported function, static postural control, and dynamic postural control.

Which of the following is true with regards to rotator cuff injuries? A. Pain is maximal with arm below breast level B. Pain is minimal between 90 and 120 degrees of active elevation/abduction C. Patients often describe crepitus D. Radiation of pain goes past the elbow E. External rotation causes a sharper stabbing pain

*C* A. Pain is *minimal* with arm below breast level B. Pain is *maximal* between 90 and 120 degrees of active elevation/abduction *C.* D. Radiation of pain *does not* go past the elbow E. *Internal* rotation will cause sharper stabbing pain

All of these are testing a similar structure except: A. Posterior drawer test B. Posterior sag test C. Hamstrings active test D. Quadriceps active test E. Reverse pivot shift test

*C. Hamstring active test*

Internal snapping (extra-articular) is often the result of: A. Labral tears B. Iliotibial band rubs over the greater trochanter C. Iliopsoas tendon moves over a bony prominence D. Iliotibial band rubs over the lesser trochanter

*C. Iliopsoas tendon moves over a bony prominence* Bony prominence can be: -iliopectineal eminence -femoral head -lesser trochanter

The most common ACL tears occur due to: A. Stretching maneuvers B. Hyper-Flexion maneuvers C. Pivoting maneuvers D. Hyper-Extension maneuvers

*C. Pivoting maneuvers* and *landing from a jump* (females)

Which of the following is false with regards to the tibiofemoral joint: A. Has a concave tibia platform B. Tibia is attached to a convex femur C. Concave tibia moves on the convex femur in the opposite direction as the physiological movement of the joint D. Concave tibia moves on the convex femur in the same direction as the physiological movement of the joint

*C.* Concave tibia moves on the convex femur in the opposite direction as the physiological movement of the joint

Which of the following can magnify shoulder symptoms: A. Salt and Vinegar chips B. Happiness C. Co-workers D. Worker's compensation

*D* Depression, worker's compensation, other insurance claims and other life stresses

Which of the following combinations is incorrect? A. Anterior Drawer Test : Integrity B. Lachman Test : Injury C. Pivot-Shift Test : Instability D. Lachman Test : Integrity E. Anterior Drawer Test : Instability

*D* and *E*

Which is the most completely disrupted ligament of the knee? A. MCL B. PCL C. LCL D. ACL

*D. ACL*

Which of the following MOI is correct for PCL injuries: A. Direct blow to the proximal fibula B. A fall on the knee with the foot in a dorsiflexed position C. Hyperextension of the knee > hyperflexion of the knee D. Combined rotational forces

*D. Combined rotational forces* A. Direct blow to the proximal *tibia* B. A fall on the knee with the foot in a *plantarflexed* position C. *Hyperflexion* of the knee > *Hyperextension* of the knee

Which statement is correct? A. If extensor lag is present, joint and muscle stiffness are likely contributors B. If extensor lag is present, joint stiffness is a likely contributor C. If extensor lag is present, muscle stiffness is a likely contributor D. If extensor lag is present, joint and muscle stiffness are not likely contributors

*D. If extensor lag is present, joint and muscle stiffness are not likely contributors*

Which of the following is included in the category hip mimickers A. Muscle strains B. Weak muscles C. Strong muscles D. Intra-abdominal disorders

*D. Intra-abdominal disorders* •Athletic pubalgia •Osteitis pubis •Genitourinary disorders •Intra-abdominal disorders •Lumbar radiculopathy

Which of the following indication is incorrect for operative treatments of MCL injuries? A. A large bony avulsion B. A concomitant tibial plateau fracture C. Associated cruciate ligament injury D. Extra-articular entrapment of the end of the ligament

*D. Intra-articular* entrapment of the end of the ligament

Which type of meniscal tears often result in locking of the knee (inability to fully straighten the knee)? A. Longitudinal meniscal tear B. Traumatic meniscal tear C. Complex meniscal tear D. Bucket handle meniscal tear

*D.* Bucket handle meniscal tear

Which MOI for MCL is incorrect? A. Direct blow to outer aspect of upper leg B. Direct blow to outer aspect of lower thigh C. Non-contact external rotation D. Non-contact internal rotation

*D.* Non-contact internal rotation

What functional exam findings are associated with PFPS?

*Decreased knee flexion* angle during jump-landing task as well as an *increased hip internal rotation angle* and *decreased vertical ground reaction force*.

Which DDX is wrong for MCL injuries? A. Patellar dislocation or subluxation B. Medial knee contusion C. Medial meniscal tear D. Aphyseal fracture in patients who are skeletally immature E. All are correct F. None are correct

*E. All are correct*

Which of the following is false with regards to MCL structure? A. The fascia investing the sartorius muscle is the first layer B. Capsule of the knee joint and the deep MCL is the third layer C. Superficial MCL, the medial patellofemoral ligament, and the ligaments of the posteromedial corner of the knee is the second layer D. Capsule of the knee joint and the meniscofemoral and meniscotibial ligaments composes the third layer E. All are correct F. None are correct

*E. All are correct*

Which of the following ACL rehabilitation protocols is/are incorrect? A. Control swelling and pain B. Early emphasis on obtaining full passive extension C. Complete reduction of intra-articular inflammation D. Early initiation of quadriceps and hamstring activity E. Use of isometric exercises

*E.* Use of *open and closed kinetic chain* exercises A, B, C and D are True - the bolded material completes the sentence A. Control swelling and pain *to limit muscular inhibition and atropy* B. *Early weightbearing and ROM with* Early emphasis on obtaining full passive extension C. Complete reduction of intra-articular inflammation *and swelling before surgery to avoid arthrofibrosis* D. Early initiation of quadriceps and hamstring activity

Several studies have indicated that more than half of those with chronic ankle instability have associated ________ conditions or injuries such as...

*Extra-articular conditions/injuries.* -Articular cartilage damage. -Peroneal tendon injuries. -Impingement lesions -Associated tarsal conditions.

Which of the following is included in extra-articular pathologies of the hip: A. Tendons B. Muscles C. Bursase D. Fascia E. Nerves F. All of the above

*F.*

T or F: Hamstring strengthening is the foundation of rehabilitation after PCL injury.

*False* *Quadriceps* strengthening is the foundation of rehabilitation after PCL

T or F: Patellofemoral pain syndrome is the most common orthopedic injury among active young men.

*False* *women*

T or F: Secondary stabilizers can be relaxed if in a 45-55 degree angle.

*False* 20-30 degree angle

T or F: A posterior shear force occurs during open kinetic chain exercises throughout the entire ROM of the knee.

*False* A posterior shear force occurs during *closed kinetic chain* exercises throughout the entire ROM of the knee.

T or F: A weak VMO and hip adductors can also increase the Q-angle

*False* A weak VMO and *hip abductors* can also increase the Q-angle when the VMO and hip abductors are weak the action of hip adduction and the un-resisted motion of vastus lateralis forces the femur and tibia to collapse medially and thus create a greater Q-angle

T or F: By definition a combined PCL and collateral complex injury will have varus and valgus laxity in full flexion.

*False* By definition a combined PCL and collateral complex injury will have varus and valgus laxity in full *extension*

T or F: Comprehensive LE muscle stretching and strengthening should be performed while muscle conditioning should be reserved for later

*False* Comprehensive LE muscle stretching and strengthening *and conditioning*

T or F: If AMI and joint effusion are present the most important goal is to increase both Active and Passive ROM as the first stage of rehabilitation efforts

*False* If AMI and joint effusion are present the most important goal is to *reduce joint effusion* as the first stage of rehabilitation efforts

T or F: if a patient has supination in weight bearing the tibia medially rotates

*False* If a patient has *pronation* in weight bearing the tibia medially rotates *and increases the Q-angle*

T or F: Isolated PCL tear causes varus-valgus laxity and decreased rotation

*False* Isolated PCL tear *does not* cause varus-valgus laxity *or increased* rotation

T or F: Collateral ligaments provide protection and stability against ONLY medial stresses of the knee

*False* Provides for both medial and lateral stresses

T or F: Severe posterolateral corner instability can occur without injury to the PCL, fibular collateral ligament and popliteus

*False* Severe posterolateral corner instability *is difficult to occur* without injury to the PCL, fibular collateral ligament and popliteus

T or F: The anterior drawer test shows anterior translation but secondary stabilizers allow translation with the knee in a 90 degree angle

*False* The anterior drawer test shows anterior translation but secondary stabilizers *will limit* translation with the knee in a 90 degree angle

T or F: The collateral ligaments are taut during lateral rotation of the tibia on the femur

*False* The collateral ligaments are taut during *medial* rotation of the tibia on the femur and *when the knee incurs varus stress*

T or F: The most sensitive PCL orthopedic test is the Posterior Sag Test

*False* The most sensitive PCL orthopedic test is the *posterior drawer test at 90 degrees of knee flexion*

T or F: Degenerative meniscal tears have a higher healing rate than traumatic meniscal tears

*False* Traumatic meniscal tears have a higher healing rate than traumatic meniscal tears

T or F: ACL contains efferent mechanoreceptors that affect the knee's stability?

*False* ACL contains *afferent* mechnoreceptors that affect the knee's stability

T or F: Lateral Collateral Ligament attaches to the lateral meniscus

*False* Does not attach

T or F: Radial meniscal tears have a more favorable healing potential compared to longitudinal meniscal tears.

*False* Longitudinal meniscal tears have a more favorable healing potential compared to radial meniscal tears.

T or F: MCL restricts medial rotation of the tibia on the femur

*False* Restricts lateral rotation

T or F: A great way to rehab the meniscus is to do wall sits at 60 degrees.

*False* wall sits at *50* degrees flexion angles greater than 60 degrees translate the menisci posteriorly which increases translation and places detrimental stresses across a healing meniscus

T or F: MOI for Acetabular labral tears are with repetitive hip extension

False - *with repetitive hip flexion*

What is the treatment for external coxa saltans (IT band syndrome)?

-*Rest* for initial inflammation, along with supplements and modalities to reduce inflammation. -Gentle stretching and strengthening (hip abduction and external rotation) once inflammation is *gone*.

What are some strategies to PCL rehabilitation?

-A posterior shear force occurs during closed kinetic chain exercise throughout the *entire* ROM of the knee, so there are greater forces generated as knee flexion increases: So *no wall sits*. -With open kinetic chain activities, there is a tremendous force exerted on the PCL during flexion exercises. -*Passive* motion can be safely performed through the entire range of flexion and extension. -Active closed kinetic chain activities of any kind, in any ROM, should be used cautiously when rehabilitating the PCL, either as nonoperative therapy or after reconstruction. -Avoid open kinetic chain flexion exercises. Start with *short arc* exercises (0-30 degrees) -Open kinetic chain extension exercises performed at lower *flexion* angles (60-0 degrees) are safe. -Non-resisted active motion exercise as tolerated is safe for grades I and II injuries. Limit resisted motion, including weightbearing to a 0-60 degree flexion arc during early treatment. -Grade III injuries should be braced in extension and slowly increased. -*Quadriceps strengthening* is the foundation of rehabilitation after *PCL injury*. -Use closed kinetic chain activities from 0-45 degrees to protect the PCL and patellofemoral joint. Open kinetic chain extension activities place the lowest strains on the PCL but result in elevated patellofemoral joint forces.

What are the possible intra-articular hip injuries?

-Acetabular labral tears -Ligamentum teres tears -Femoroacetabular impingement -Chondral defects -Osteoarthritis -Osteonecrosis -Dysplasia

What are some ACL rehabilitation guidelines?

-Achieve full ROM and complete reduction of intra-articular inflammation and swelling before surgery to avoid arthrofibrosis. -Early weightbearing and ROM, with *early emphasis on obtaining full passive extension.* -Early initiation of quadriceps and hamstring activity. -Control swelling and pain to limit muscular inhibition and atrophy. -Appropriate use of open and closed kinetic chain exercises. -Comprehensive lower extremity muscle stretching, strengthening and conditioning. -Neuromuscular and proprioception retraining including perturbation training. -*Stepped progression based on achievement of therapeutic goals.* -Functional testing and functional sport-specific training prior to return to play.

What is conservative treatment for adductor strains?

-Generally, initial treatment includes physical therapy modalities such as rest, ice, compression, and elevation to help prevent further injury. -Restoration of ROM and prevention of atrophy. -Focuses on regaining strength, flexibility, and endurance.

T or F: another rehab treatment for mallet finger is to work on AROM of both DIP and PIP

False - AROM of MCP and PIP

What is conservative treatment for internal snapping hip?

-Address asymmetries in muscular flexibility and deficits in core strength. =*From Johnston et al:* -Daily external and internal hip rotator strengthening in sitting (with hip flexed to 90 degrees) for three sets of 20 repetitions. ( 2 weeks daily). -Progressed to side-lying abduction/external rotation with the knees flexed to 90 degrees and the hip flexed to 45 degrees (clam-shells) 3 sets of 20 daily. -The individual stands against a wall for support while performing a single-leg mini-squat on the affected side. During the mini-squat, the knee must remain over the lateral foot to keep the hip in external rotation. 2-3 times per week 2 sets of 20. -Perform daily stretching of the iliopsoas. =*From Gruen et al:* -Reported a *63%* success rate with a 3-month program training program that included iliopsoas stretching, concentric strengthening of the external and internal hip rotators, and eccentric strengthening of the hip flexors and extensors.

Holmich's (2007) study of 207 athletes with groin pain identified several dysfunctions as the cause...

-Adductor-related dysfunction (58%) was the primary clinical entity. -Iliopsoas dysfunction (36%). -Rectus abdominus-related dysfunction (6%). -Multiple causes found in 33%.

What are some exercises that will help re-establish ROM for ankle sprains?

-Ankle pumps -Plantarflexion and dorsiflexion ROM progressing from passive ROM to active-assisted ROM (AAROM) to active ROM as tolerated. -Heel-cord and posterior calf stretches. -ABCs or alphabets. -Towel curls and/or marble pick-ups.

What are the DDXs for hip injuries (hip mimickers)?

-Athletic pubalgia (sports hernia). -Osteitis pubis -Cenitourinary disorders -Intra-abdominal disorders. -Lumbar radiculopathy

What conditions can cause diffuse heel pain?

-Calcaneal stress fracture. -Calcaneal fracture

What is the level B recommendation (moderate evidence) to treating plantar fasciitis?

-Calf muscle and/or plantar fascia-specific stretching can provide short-term (2-4 weeks) pain relief and improved function. The dose for calf stretching can be either 3 times a day or 2 times a day utilizing either a sustained (3 minute) or intermittent (20 seconds) stretching time, as neither dosage produced a statistically significant better effect. -Night splints (1-3 months) should be considered for patients with symptoms lasting more than 6 months. The desired length of time wearing the splints is 1 to 3 months. The type of night splint (posterior, anterior, or sock type) does not appear to affect outcome.

What sensory motor exercises should you use to rehab ankle sprains?

-Circling the board while touching all sides of the board in both clockwise and counterclockwise directions. -Towel curls -Marble pick-ups

What are the goals of phase 3 MCL rehab?

-Completion of a running program. -Completion of a series of sport-specific activities.

What characteristics of meniscal tear indicate non-favorable healing potential?

-Complex -Degenerative -Chronic

What therapies may be used in phase 1 MCL rehab?

-Cryotherapy -Therapeutic cold is applied to the medial aspect of the knee for 20 minutes every 3-4 hours for the first 48 hours. -Early cryotherapy provides anesthesia and local vasoconstriction to minimize initial hemorrhage and reduce secondary edema. Leg elevation also helps limit swelling.

What are the functions of the meniscus?

-Cushion the joint. -Deepen the socket. -Increase joint congruity to better distribute weight-bearing forces. -Assist in joint lubrication. -Provide stability.

What does it mean if AMI persists?

-Decrease in stability of the knee. -Decrease of function or performance. -Increase in injury.

What mechanisms of injury are associated with MCL disruption?

-Direct blow to the outer aspect of the upper leg or lower thigh creating a valgus force. -Noncontact external rotation mechanisms can cause MCL injury, but also typically result in associated injury, usually involving the cruciate ligaments.

What mechanisms of injury can affect the PCL?

-Direct blow to the proximal tibia. -A fall on the knee with the foot in a plantarflexed position. -Hyperflexion of the knee.

What neuromuscular re-education exercises should be used to rehab ankle sprains?

-Early exercises to encourage loading of the ankle include "weight shifts" in various directions. -Weight shifts: the patient stands with his or her weight shifted to the non-injured leg, then progressively shifts the weight onto the injured leg before returning to the NWB position. -Patient should perform these step-ups and step-downs in various directions. -Patient-controlled perturbations. -Patient can also perform these activities in a tandem stance or a single-legged stance. -Doctor initiated perturbations.

________ is the most common cause of snapping in the hip. This cause is aka ________.

-External coxa saltans. -Aka IT band syndrome.

When is a brace used in phase 1 MCL rehab?

-For grades 2 and 3 sprains, a lightweight hinged brace is worn. -The brace should protect against valgus stresses of daily living but should not restrict motion or inhibit muscle function. -The brace is worn at all times except for bathing during the initial 3-4 weeks. -Use of knee immobilizers and full-leg braces is discouraged because they tend to inhibit motion and prolong the period of disability.

What patellofemoral compressive forces are created during the weight-bearing phase of gait? Stair climbing? Squatting?

-Gait: 1.5 x bodyweight. -Stairs: 3 x bodyweight. -Squatting: 7 x bodyweight.

What strength training exercises should be used to rehab ankle sprains?

-Isometric exercises should begin with submaximal contractions and progress to maximal contractions. -Isometric exercises should be progressed to isotonic exercises as tolerated. Resistance can be provided manually, with cuff weights or elastic bands or cords. -Isotonic exercises should begin with a limited ROM and progress to full ROM as tolerated and should progress from submaximal resistance to maximal efforts. -As weightbearing becomes tolerated, heel and toe raises can be incorporated as can walking on the heels or toes. -As the patient's pain-free ROM increases, proprioceptive neuromuscular facilitation (PNF) techniques can be used.

What is the function of the tibiofemoral joint capsule?

-Joint stability -Moves synovial fluid around the joint and is connected with bursae. -Mechanoreceptors (ruffini nerve endings) are sensitive to pressure and deformation.

What usually causes intra-articular coxa saltans?

-Labral tear -Chondral damage -Loose bodies (cartilage or osteophyte broke off). -Other pathology within hip joint.

What are the indications for operative treatment of an MCL injury?

-Large bony avulsion. -Concomitant tibial plateau fracture. -Associated cruciate ligament injury. -Intra-articular entrapment of the end of the ligament.

What structures are attached to the medial meniscus?

-MCL -ACL -Semimembranosus tendon

What are some MCL injury DDXs?

-Medial knee contusion. -Medial meniscal tear. -Patellar dislocation or subluxation. -Apophyseal fracture in patients who are skeletally immature.

What are the clinical signs and symptoms of calcaneus stress fracture?

-Much more common in athletes and runners with overuse history and repetitive high-impact activity or elderly females with osteoporosis and overuse in their walking or exercise regimen. -Pain is more *diffuse* than plantar fasciitis, with a positive squeeze test -*If they can pinpoint the pain then it is NOT a stress fracture.*

What are the phase 1 goals for MCL rehab?

-Normal gait. -Minimal swelling. -Full ROM. -Baseline quadriceps control.

What are risk factors for plantar fasciitis?

-Overuse secondary to work-related prolonged weightbearing. -Unaccustomed running or walking. -Inappropriate shoe wear. -Increased BMI. -Limited ankle dorsiflexion.

When you have severe posterolateral corner instability, what other structures will most likely be involved?

-PCL -Fibular collateral ligament -Popliteus

What are the clinical signs and symptoms of plantar fasciitis?

-Pain and tenderness located inferiorly at the plantar fascia origin (not posteriorly). -Almost all patients complain of inferior heel pain in the mornings with the first few steps and may complain of pain after prolonged walking or standing.

What are the clinical signs and symptoms of posterior tibial tendon (PTT) insufficiency?

-Pain is medial rather than inferior or posterior. Often, difficulty or inability to do a unilateral heel raise. -Point tender and boggy along course of PTT medially.

What are the clinical signs and symptoms of achilles tendinitis?

-Pain is posterior rather than inferior. Haglund's deformity (pump bump) is tender over prominent bony deformity and often rubs or is irritated by the heel box of the shoe. -Complete rupture of the Achilles tendon describe a feeling of being "shot" in the tendon while pushing off, have a positive Thompson squeeze test, and have a lack of active plantar flexion except a small flicker from the long toe flexors.

What clinical findings are associated with acetabular labral tears?

-Painful clicking and locking or giving way in the hip. -Pain increases with hip extension (when the anterior labrum is torn).

What are the clinical signs of a Grade II ankle sprain?

-Partial tearing of ligaments, usually the ATFL and CFL. -Point and diffuse tenderness. -Moderate dysfunction. -Slight to moderate laxity. -Antalgic gait and pain with full weight-bearing, may need supportive device to ambulate. -Mild to moderate edema.

What mechanical factors contribute to chronic ankle instability?

-Pathologic laxity -Arthrokinetic restriction -Synovial changes -Degenerative changes

What conditions can cause lateral heel pain?

-Peroneal tendon disorders. -Lateral calcaneal nerve neuritis.

What are the symptoms of chronic ankle instability?

-Persistent pain -Recurrent sprains -Repeated episodes of the ankle giving way.

What are the 3 types of hip impingement?

-Pincer -Cam -Combined

What conditions can cause plantar heel pain?

-Plantar fasciitis, plantar fascial rupture, partial plantar fascial rupture. -Calcaneal spur or heel spur. -Fat pad syndrome -Calcaneal periostitis. -Compression of the nerve to the abductor digiti quinti. -Calcaneal apophysitis

What structures are attached to the lateral meniscus?

-Popliteus -PCL

What orthopedic tests are assessing PCL integrity?

-Posterior drawer -Posterior sag -Quadriceps active test -Reverse pivot shift test

What conditions can cause medial heel pain?

-Posterior tibial tendon disorders (insufficiency, tenosynovitis, rupture). -Tarsal tunnel syndrome. -Jogger's foot (medial plantar neuropraxia). -Medial calcaneal neuritis.

What conditions can cause posterior heel pain?

-Retrocalcaneal bursitis. -Haglung's deformity (pump bump). -Calcaneal exostosis -Tendinoachilles tendinitis

Which of the following are acceptable rehab modalities for De Quervain Tenosynovitis: A. Soft Tissue Mobilization B. Ultrasound C. Cold laser D. Contrast baths E. TENS

-ST mobilization -US -Iontophoresis w/ topical steroid -ice massage -contrast baths

What characteristics of meniscal tear indicate favorable healing potential?

-Simple -Traumatic -Acute

What tests are used for the evaluation of syndesmosis injuries?

-Squeeze test -External rotation test -Fibula translation test -Cotton test -Crossed leg test

What are the steps to treating and rehabilitating ankle sprains?

-Step 1: Protect the area from further injury. Except for Grade I, where you want them to keep moving. -Step 2: Decrease pain, swelling, and spasm. -Step 3: Re-establish range of motion (ROM), flexibility, and tissue mobility. -Step 4: Re-establish neuromuscular control, muscular strength, endurance, and power. -Step 5: Re-establish proprioception, coordination, and agility. -Step 6: Re-establish functional skills.

What are the clinical signs of a Grade I ankle sprain?

-Stretching of ligaments, usually the ATFL. -Point tenderness. -Limited dysfunction. -No laxity. -Able to bear full weight. -Little to no edema.

What are the clinical signs of a Grade III ankle sprain?

-Substantial tearing of ligaments, may involve the PTFL in addition to the ATFL and CFL. -Point and diffuse tenderness. -Moderate to severe dysfunction. -Moderate to severe laxity. -Limited to no ability for full weight-bearing without supportive device. -Severe edema: edematous pocket.

What other conditions can cause heel pain?

-Systemic disorders (often bilateral heel pain). -Reiter's syndrome -Ankylosing spondylitis -Lupus -Gouty arthropathy -Pseudogout -Rheumatoid arthritis -Systemic lupus erythematosus

What is the clinical presentation of someone with a gluteal tear?

-Tender to palpation over the greater trochanter. -"Grinding" sensation along with pain when lying on the affected side and with single-leg stance activities such as climbing stairs. -Hip abductor weakness. -Pain during passive and resisted external rotation with the hip flexed to 90 degrees and with single-leg stance for more than 30 seconds.

What can cause gluteal tears?

-Tendonitis can lead to tendon thickening and progress to partial and then complete tearing of both the gluteus medius and minimus. -Excessive tensioning of the IT band over the greater trochanter (putting excessive pressure on the gluteal tendons?).

If the patient has pronation in weight bearing, what happens to the tibia and Q-angle? Lateral rotation?

-The tibia medially rotates and increases the Q angle. -With lateral rotation, the Q angle decreases.

What orthopedic tests may be positive with gluteal tears?

-Trendelenburg: may not even be able to stand on a single leg. -Trendelenburg gait: leaning over the involved lower extremity during the stance phase and causing passive abduction of the hip to decrease load to the gluteals.

What will the Anterior Draw test show?

-Will show anterior translation of the knee. -Secondary stabilizers will limit translation with the knee in a 90 degree angle.

Recurrence of trigger finger after injection can occur in the following groups

-younger age -insulin dependent DM -those involving multiple fingers

Improvement following an elbow dislocation lasts ________.

3-5 months

Seated/standing rows works the lower trap ________ more than upper trap.

3.3x

Valgus stress testing of the MCP is normally done at what angle?

30 degrees of joint flexion

What degrees would indicate a complete rupture to the UCL?

30-35 degrees

In order to give soft tissue a rest, elbow dislocation should be splinted for _________.

5-7 days

Mallet Finger: stack splints should be used for ____ weeks

6-10 weeks to immobilize the DIP

In the Green et al article - what percentage was reported to be a success with a 3-month treatment protocol?

63%

Trigger finger treatments: corticosteroid and lidocaine injections have __% improvement from one injection

66% *75-85% for multiple injections

How is PCL tear best detected?

70-90 degrees of knee flexion with posterior drawer testing.

What is the direction an elbow dislocation is most likely to go.

90% are posterior or posterolateral

____% of issues at the knee are not forming from the knee. Usually they are from the hip or foot.

95%

What is arthrogenic muscle inhibition (AMI)?

A protective reflex inhibition to protect the knee from a strong quadriceps contraction. It knows that it is injured and therefore it is going to deregulate a lot to prevent further injury.

T or F: the Ulnar Collateral Ligament provides resistance to ulnar applied forces

False - UCL provides radially applied forces

The most unstable position of the shoulder is: A. 90 degrees of abduction and external rotation B. 90 degrees of adduction and external rotation C. 90 degrees of abduction and internal rotation D. 90 degrees of adduction and internal rotation

A. 90 degrees of abduction and external rotation

T or F: a torn UCL weakens pinch grip strength and allows volatility subluxation of the distal phalanx

False - _______ of the proximal phalanx

Which of the statements is false with regards to correcting Internal snapping hip as suggested by Johnston et al (1998)? A. External and internal hip rotator strengthening in sitting position B. Clam shells C. Daily stretching to psoas D. Single leg mini squats on non-affected side

D. Single leg mini squats on *affected* side

During flexion, the menisci are pulled posteriorly by the ________ and ________. During extension they are pulled anteriorly by the ________.

During flexion, the menisci are pulled posteriorly by the *semimembranosus* and *popliteus*. During extension they are pulled anteriorly by the *meniscopatellar ligaments*.

When should you introduce sensory motor exercises for ankle-sprain rehab?

During the subacute phase. Keep motions slow and controlled at all times. -Begin with dorsiflexion, plantarflexion, and eversion before incorporating inversion.

Retraction of the FDP to the palm describes: A. Type 1 Jersey Finger B. Type 2 Jersey Finger C. Type 3 Jersey Finger

A. Type 1

T or F: after pain control and regaining shoulder motion rehab should focus on re-educating muscles through learned motor patterns

False - after pain control.....*rehab should focus on strengthening and re-educating the muscles around the shoulder to perform their normal tasks*

What is the most frequently completely disrupted ligament in the knee?

ACL

The CFL is usually injured in combination with the ________.

ATFL

About ____% of sports-related ACL tears are ________ injuries, occurring during pivoting maneuvers or landing from a jump.

About *80%* of sports-related ACL tears are *noncontact* injuries, occurring during pivoting maneuvers or landing from a jump.

What occurs in femoracetabular impingement?

Acetabulum and femur contact each other prematurely during hip motion

T or F: ganglion cysts present with a mass ONLY on the dorsal aspect of the wrist

False - both dorsal and volar aspect

PFPS remains the most common orthopedic injury among ________ (demographic).

Active young women.

What are some non-surgical treatment for post traumatic elbow stiffness?

Adjuncts to this therapy may include non steroidal anti-inflammatory drugs (NSAIDs), heat or ice application, and therapy modalities such as massage, iontophoresis, ultrasound, and electrical stimulation. Static progressive adjustable splints such as the turnbuckle splint ( used for flexion-extension deficits), supination-pronation splints (most effective when used for patients with symptoms of less than 6 months to 1 year with little articular involvement.

Initial stretching of the IT band with external coxa saltans treatment will ________ the condition.

Aggravate

What is coxa saltans? Aka?

Aka snapping hip. There is audible "snapping" heard by the patient and therapist of palpable to the therapist. -Hip pain during activity.

The ACL and PCL provide ________ and ________ stability.

Anterior-posterior and rotational stability.

What is Cam impingement?

Anterosuperior aspect of the femoral head-neck junction is aspherical or flattened or has a convex contour. -Flexion with rotation will impinge the labrum (as in squatting).

T or F: ganglion cysts present with pain with ONLY full extension of the wrist

False - full flexion or extension

T or F: hallmark finding of a mallet finger is an extended posture of the DP joint

False - is a flexed (or drooped posture) of the DIP joint

What is the gold standard for TFC Injury diagnosis?

Arthroscopy

Who tends to get internal snapping hip (extra-articular coxa saltans)?

Athletes whose sport requires frequent hip movements into high flexion angles combined with additional external and internal hip rotation. -Ex// Jumping track athletes, figure skaters, gymnasts, martial arts.

Half of all ankle sprains occur during ________ activity.

Athletic

T or F: Structural injury to rotator cuff is most characteristic in active young adults under the age of 40

False - most characteristic in those older than 40 years of age

T or F: Frozen shoulder is often cause by a traumatic injury

False - not caused by trauma

T or F: anyone older than 40 with shoulder pain can be presumed to have a rotator cuff tear until proven otherwise

False - older than 60 years of age

T or F: the only key features of UCL rupture is pain and swelling

False - pain, swelling and ecchymosis at the ulnar aspect of the thumb base

T or F: mallet finger produces a flexor lag at the DIP joint

False - produces an extensor lag at the DIP joint

T or F: the scapula is not mobile on the thoracic wall

False - the scapula is *very* mobile on the thoracic wall

T or F: the key features of UCL rupture occurs at the ulnar aspect of the index base

False - ulnar aspect of the thumb base

T or F: men are more common to be affected with carpal tunnel syndrome

False - women are more common

T or F: treatment of intra-articular pathology via surgical methods aims to increase blood supply in the hip which will increase the success of conservative management

False -*decrease* blood supply in the hip which will *reduce* the success of conservative management

The middle GH ligament imparts stability against anterior translation with the arm in: A. External rotation B. External rotation and abduction < 90 degrees C. External rotation and adduction < 90 degrees D. External rotation and adduction < 90 degrees

B. External rotation and abduction < 90 degrees

Which of the following is correct for calcified tendonitis A. Young age B. Middle age C. Old age D. Advanced age

B. Middle age

T or F: strengthening of scapular stabilizers should be the last portion of shoulder rehab

False -*strengtening of scapular stabilizers is important early on in the rehabilitation program of shoulders*

T or F: perform SLI adjustments on nodule while pulling finger into flexion

False —> on extension

True/false: A combination of PCL tear and posterolateral corner tear produces much less severe posterior translation and external rotation than either injury in isolation.

False. It produces much *more* severe posterior translation and external rotation when both are prosent.

T or F: If extensor lag is not present after 6 weeks DO NOT NIGHT SPLINT

False: if there is no extensor lag after 6 weeks, night splint for 3 more weeks and splint with sports/activities

Retraction to the proximal interphalangeal (PIP) joint describes: A. Type 1 Jersey Finger B. Type 2 Jersey Finger C. Type 3 Jersey Finger

B. Type 2

What is the significance of the MCL attaching to the medial meniscus?

Because they are connected, they get injured together.

A hard end feel at the extremes of motion indicates _______________ & a soft end feel indicates ____________.

Bony constraint Soft tissue constraint

Compression neuropathy of the ulnar digital nerve of the thumb describes what

Bowler's Thumb

Digital Nerve Injury is an aka for what condition

Bowler's Thumb

Painful mass and paresthesia at the base of thumb describes what condition

Bowler's Thumb

Bucket handle tears often result in ________ of the knee when the bucket handle displaces toward the _________.

Bucket handle tears often result in *locking* of the knee when the bucket handle displaces toward the *midline*.

Poor prognosis for mallet finger includes which: A. People over the age of 40 B. Treatment must be sought within 4 weeks C. Initial extensor lag is greater than 50 degrees D. Short stubby fingers E. Peripheral vascular disease or associated arthritis

C, D and E are ALL TRUE A and B are FALSE because: A. Over age 60 B. Poor prognosis when treatment sought after 4 weeks

The meniscus has all of the following characteristics except: A. Cushions the joint B. Deepens the socket C. Resists joint lubribation D. Provides stability

C. *Assist* in joint lubrication -cushion the joint -deepens the socket -increase joint congruity to better distribute weight-bearing forces -assist in joint lubrication -provide stability

Which of the following clinical presentations of external coxa saltans is incorrect? A. Iliotibial band (IT band) rubs over the greater trochanter causing inflammation B. Palpation over the greater trochanter may lead to an increase in symptoms C. Negative Ober's test D. Pain is experienced when an individual is lying on the affected side

C. *Positive* Ober's test

The greater ground reaction force (landing on less flexed knees), the greater the likelihood for _________.

Osteoarthritis

Which of the following is included in intra-articular hip injuries? A. Piriformis syndrome B. SCFE C. Dysplasia D. Genitourinary disorders

C. Dysplasia •Acetabular labrum tears •Ligamentum teres tears •Femoraacetabular impingement •Chondral defects •Osteoarthritis •Osteonecrosis •Dysplasia

The most important anterior stabilizer of the shoulder is: A. Superior GH ligament B. Middle GH ligament C. Inferior GH ligament D. All of the above

C. Inferior GH ligament

Which of the following statements is/are true: A. An anterior/Inferior laxity of the GH can be observed in an external rotation with or without adduction B. Posterior instabilty is seen when a patient experiences pain during the follow through with the arm in a forward flexion/abduction/internal rotation C. Pain associated to inferior distraction forces suggests inferior capsular laxity D. Pain associated to superior distraction forces suggests a multidirectional instability

C. Pain associated to inferior distraction forces suggests inferior capsular laxity A. An anterior/Inferior laxity of the GH can be observed in an external rotation with or without *abduction* B. Posterior instabilty is seen when a patient experiences pain during the follow through with the arm in a forward flexion/*adduction*/internal rotation D. Pain associated to *inferior* distraction forces suggests a multidirectional instability

Bony fragment distal to the A4 pulley describes: A. Type 1 Jersey Finger B. Type 2 Jersey Finger C. Type 3 Jersey Finger

C. Type 3

What two malalignment problems should be investigated in patients with patellofemoral pain?

Compensatory pronation and hip medial rotation with resulting medial tibial rotation.

What does the following describe: due to the bone spurs on the femoral head flexion with rotation will impinge the labrum

Cam impingement

What usually causes extra-articular coxa saltans?

Can be either *internal or external*. -Internal snapping is caused by movement of the *iliopsoas* tendon over a bony prominence. -External snapping is caused by movement of the *iliotibial band* rubbing over the greater trochanter. Excessive friction causes bursae to become inflamed.

What effect does a weak vastus medialis oblique (WMO) and hip abductors have on Q angle?

Can increase it.

What is the most common peripheral neuropathy?

Carpal Tunnel Syndrome

Tendinosis sx

Chronic degeneration, regeneration & microtears May be painful

Clinically, the ________ MCL is ruptured more frequently than the ________ MCL.

Clinically, the *deep* MCL is ruptured more frequently than the *superficial* MCL.

Trigger finger may be accompanied by

Clinking, locking or popping

What bracing is used in phase 3 MCL rehab?

Continued use of the brace is recommended during this phase and for the rest of the athletic season. This may protect against further injury and at least provides phychologic support.

What bracing is used in phase 2 MCL rehab?

Continued use of the lightweight hinged brace.

What is the chief complaint of someone with extra-articular snapping?

Painful, repetitive clicking and sometimes "clunking" in the anterior hip (mostly superficial when you feel it). -Passive movement from hip flexion and external rotation to extension with internal rotation hurts a lot.

Requirements for Safe Return to Play

Painless & full ROM (with resistance) No tenderness Satisfactory isokinetic mm strength testing Satisfactory clinical exam

When is it safe for a recently injured athlete to return to work.

Painless and full ROM. No tenderness Satisfactory isokinetic muscular strength testing Satisfactory clinical examination

Which treatment protocol is appropriate for TFC injuries: A. 4-6 week brace —> aggressive motion exercises —> resisted strength rehabilitation —> active assisted and passive ROM B. 4-6 week brace —> aggressive motion exercises —> active assisted and passive ROM —> resisted strength rehabilitation C. Active assisted and passive ROM —> 4-6 week brace —> aggressive motion exercises —> resisted strength rehabilitation D. 4-6 week brace —> active assisted and passive ROM —> aggressive motion exercises —> resisted strength rehabilitation

D. 4-6 week brace —> active assisted and passive ROM —> aggressive motion exercises —> resisted strength rehabilitation

Which of the following descriptors is false with regards to gluteal tears? A. Tendonitis —> tendon thickening —> partial/complete tear of gluteus medius and minimus B. Occurs through 4th-6th decades of life C. More common in women D. Excessive tensioning of teh IT band over the lesser trochanter

D. Excessive tensioning of the IT band over the *greater trochanter*

What is the clinical presentation for an MCL/ulnar nerve injury?

Hypertrophied flexor-pronator mass (often accompanies ulnar nerve parestgesia). 4-5 digit numbness or tingling and potmedial elbow pain.

How long does it take to rehab a Grade I ankle sprain? II? III?

I: 1-2 weeks. II: 4-8 weeks. III: 12-16 weeks.

Trigger finger presents with _____

Palpable nodule on flexor tendon

What composes the triangular fibrocartilage complex composed of:

Lunate Triquetrum Hamate Base of the 5th metacarpal Extensor carpi ulnaris subsheath

The ACL contains afferent ________ that affect the knee's stability.

Mechanoreceptors.

Elbow Dislocation is the m/c dislocation in? a. over the age of 10 b. under the age of 10 c. over the age of 20 d. under the age of 20

b. under the age of 10

MCL is the primary restraint to a. varus stress b. valgus stress

b. valgus stress

Coxa saltans can be either ________ or ________.

Coxa saltans can be either *intra-articular* or *extra-articular*.

How are crutches used in phase 1 MCL rehab?

Crutches are used until the patient ambulates without a limp, which takes approximately 1 week.

Which of the following is correct for biceps tendinosis A. Young age B. Middle age C. Old age D. Advanced age

D. Advanced age *who had rotator cuff disorders*

90% of elbow dislocations are? a. anterior or anterolateral b. anterior or anteromedial c. posterior or posterolateral d. posterior or posteromedial

c. posterior or posterolateral

Match the elbow dislocation rehab with the stage: At 6 to 8 weeks a. elbow is splinted to allow soft tissue rest b. a hinged elbow brace from 30 to 90 degrees is applied and active ROM is initiated c. strengthening

c. strengthening

Which statement is incorrect with PCL grade injuries and rehab? A. Grade 1 injuries: perform non resisted active motion as tolerated but resisted motion should be limited to a 0-60 degree flexion arc during the early treatment B. Grade 2 injuries: perform non resisted active motion as tolerated but resisted motion should be limited to a 0-60 degree flexion arc during the early treatment C. Grade 3 injuries: should be braced in extension and slowly increased D. Grade 3 injuries: should be braced in flexion and slowly increased

D. Grade 3 injuries: should be braced in *extension* and slowly increased

Post Traumatic Elbow Stiffness Match the following with the Type: nondisplaced articular fracture with soft tissue contracture a. type 1 b. type 2 c. type 3 d. type 4 e. type 5

c. type 3

Which of the following is false with regards to SLAP lesions: A. Produces click or catching sensation B. Produces secondary rotator cuff symptoms C. Usually results from a trauma - FOOSH D. Pain is described to be located in the anterior shoulder

D. Pain is described to be located in the *posterior* shoulder

Which of the following is included in extra-articular hip injuries? A. Sports hernia B. Lumbar radiculopathy C. Ligamentum teres D. Piriformis syndrome

D. Piriformis syndrome •Internal coxa saltans •External coxa saltans •Gluteal tears •Muscle strains •Piriformis syndrome •SCFE •Fractures

A patient with a MCL injury would present with a history of _______ elbow pain with ______ during ____ cocking phase and ______ acceleration

chronic throwing late early

Sx of detachment of superior glenoid labrum

Deep pain @ posterior shoulder Click or catching sensation Secondary rotator cuff sx or association with hx of suggestive GH instability Trauma

Which of the following should you find out during history of a GH instabilty? A. Age B. Duration of pain C. Location of pain D. Type of pain E. Direction of instability

E. Direction of instability And: degree of instability and onset

T or F: All of the following are acceptable for chiropractic intervention/rehab for trigger finger: -SLI -MRT -Ultrasound -AROM -Heat therapy

False Not heat therapy

Heterotrophic ossification occurs with _______________, ______________, _____________, or __________________.

Direct trauma Neural axis injury Surgical intervention Forceful passive manipulation

Open chain exercises are when the ___________________ is not stabilized, increasing the potential for ______________ at the GH joint.

Distal Extremity Sheared forces

Which of the following is false with regards to rotator cuff injuries: A. MOI of injury for the elderly are falls B. Night pain C. Prevents sleep D. Awakens patient from sleep E. Adduction/External rotation is a mechanism often associated with rotator cuff injuries

E. *Abduction/external rotation mechanism*

T or F: 10 degrees or more difference in shoulder internal rotation is a precursor to shoulder pathology

False - *20 degrees or more*

T or F: Passive ROM is gathered before Active ROM

False - *Active before Passive*

All of the following are clinical symptoms of carpal tunnel syndrome except: A. P/T/N in median nerve distribution B. Pain and paresthesia in palmar aspect of the hand C. Hemodialysis D. Aggravation of symptoms while holding/gripping on to something E. All are correct

E. All are correct

Normal shoulder function requires movement of which of the following: A. Sternoclavicular B. Acromioclavicular C. Glenohumeral D. Scapulothoracic articulation E. All of the above

E. All of the above and *motion interface between rotator cuff and the overlying coraacromial arch*

T or F: Inspection is the most important factor for establishing success of shoulder rehab

False - *HISTORY* is the most important

T or F: the most common direction of GH instability is superior/posterior

False - *MC direction is Anterior/Inferior*

T or F: Rotator cuff instability is the MC underlying pathology producing shoulder symptoms in patients younger than 30 years of age

False - *MC underlying pathology producing shoulder symptoms in patient's younger than 30 yoa is Glenohumeral Instability*

T or F: rotator cuff injuries are often produced by either a traumatic event or acute overuse with an adduction/internal rotation mechanism

False - *abduction/external*

T or F: in a study of 207 athletes with groin pain the most common dysfunction was an iliopsoas related dysfunction

False - *adductor related dysfunction* 1. Adductor related 2. Iliopsoas related 3. Rectus abdominus

True or False: the large humeral head that articulates with the small glenoid socket restricts motion at the expense of stability

False - *allows extremes of motion at the expense of the stability*

T or F: Femoroacetabular impingement occurs in the posterior and inferior aspect of the hip

False - *anterior and superior aspect of the hip*

T or F: Younger patients with cuff symptoms - tendinosis - their main issues are often a structural one.

False - *are often a secondary manifestation of occult primary pathology*

T or F: Internal coxa saltans is the most common cause of snapping in the hip

False - *external coxa saltans*

Rehabilitation for simple dislocation includes early _____________ which prevents _______________ & _____________.

Early active ROM Post-traumatic stiffness Favorable results

What are rehabilitation considerations for elbow dislocations?

Early active ROM is the key to preventing post-traumatic stiffness and obtaining a favorable result. Elbow is splinted for 5-7 days to allow soft tissue rest. 5-7 days after injury, a hinged elbow brace from 30-90 degrees is applied and active ROM is initiated. Active ROM is required for muscle activation and assist with elbow stability and compression across the joint. Hinged elbow brace 10-15 degrees per week. Extension returns more slowly and may continue to improve for 3-5 months. Forced terminal extension should be avoided. At 6-8 weeks, strengthening can begin.

An increased rate of AMI can be associated with joint _________.

Effusion

Clinical presentation of lateral/medial epicondylitis

Evaluate entire UE kinetic chain Decreased bilateral symmetry Decreased F/E with cubitus valgus deformity

What are clinical finding for lateral and medial epicondylitis?

Evaluate the entire kinetic chain of the upper extremity. Lack of bilateral symmetry in the upper extremities. Decreased motion in flexion and extension with a cubits valgus deformity. Tinel test for ulnar nerve irritation. Valgus stress test for UCL integrity. Varus stress test for LUCL integrity. Valgus overpressure test for posterior elbow pain for posterior,dial osteophytes

The less likely mechanism of ________ may cause injury to the deltoid ligament complex (DLC).

Eversion

T or F: biceps tendinosis is described as pain in bicep that is referred to anterior shoulder and is exacerbated with shoulder rotation

False - *forearm rotation*

T or F: gluteal tears will present with tenderness to palpation over the lesser trochanter

False - *greater trochanter*

T or F: the superior GH ligament is an important superior stabilizer

False - *inferior stabilizer*

T or F: The chief complaint of external snapping hip (extra-articular) is a painful repetitive clicking and sometimes "clunking"

False - *internal*

T or F: Muscle strengthening exercises for shoulder rehab should start with open chain kinetic exercises followed by isometric exercises

False - *isometrics —> closed chain —> open chain*

T or F: Acetabular Labral Tears are often associated with femoroacetabular impingement when the knee is extended and rotation occurs in the hip

False - *knee is flexed*

T or F: Calcified tendonitis is described as insidious, rapid, severe onset of posterior shoulder pain

False - *lateral*

T or F: raising the arm is often more painful than lowering it

False - *lowering the arm is often more painful than raising it*

T or F: the least functional of all open chain exercises are plyometric exercises

False - *most functional*

T or F: Surgical management of intra-articular pathology will help protect the damaged structures and reduce the acute symptoms

False - *non surgical*

T or F: primary focus for shoulder rehabilation is pain control and strengthening muscles around the shoulder

False - *primary focus for shoulder rehabilation is pain control and regaining the coordinated motion throughout all components of the shoulder complex*

T or F: pain opening jars, turning doorknobs or working a screwdriver are related to carpal tunnel syndrome

False - *related to De Quervain Tenosynovitis*

T or F: scapulothoracic dyskinesia can cause primary rotation to the rotator cuff

False - *secondary*

T or F: the middle GH ligament imparts stability against both posterior and anterior translation

False - *stability against anterior translation*

T or F: acute groin injuries are usually traumatic in origin

False - *usually musculoskeletal*

What tendons are involved in lateral epicondylitis?

Extensor Capri radial is breves tendon. Approximately 1/3 of cases involve extensor communist tendon. Extensor Capri radial is longus and extensor carpi ulnaris can also be involved.

Tendons attaching at lateral epicondylitis

Extensor carpi radialis brevis Extensor carpi radialis Longus Extensor carpi ulnaris Extensors digitorum

1/3 of lateral epicondylitis cases involve the ______________.

Extensor digitorum

What is the most common mechanism of injury for syndesmotic sprains?

External rotation of the foot relative to the tibia. Or eversion of the talus within the ankle mortise and excessive dorsiflexion. -Found in high contact sports or sports that require cutting or pivoting.

The large humeral head articulating with a small glenoid socket allows _________________ at the expense of ___________.

Extremes of motion Stability

Elbow dislocation 3 main MOI's

FOOSH with the arm abducted MVA Trauma/Sports injury

What is the MOI for an elbow dislocation?

Fall on an outstretched hand with the arm abducted.

T or F: NSAIDs are the most effective treatment for De Quervain Tenosynovitis

False

T or F: The lateral portions of the meniscus are highly vascularized

False

True/false: The lateral collateral ligament (LCL) attaches to the lateral meniscus.

False

T or F: shoulders funnel forces to muscles with finer motor skills such as the legs and trunks

False *shoulders secondary function is to act as a funnel to which forces from the legs and trunk are passed to muscles of the arm, forearm and hand which have finer motor skills*

T or F: During weight bearing movements from flexion into extension the femur and tibia are supported primarily by the posterior menisci

False During weight bearing movements from flexion into extension the femur and tibia are supported primarily by the *anterior* menisci

T or F: Extensor lag may occur because of edema, pain and redness

False Extensor lag may occur because of *edema, pain or weakness*

T or F: another descriptor of jersey finger is the lack of active extension of the PIP joint

False Lack of active flexion of the DIP joint

T or F: passive motion is not affected if joint or muscle stiffness is present

False passive motion is *affected* if joint or muscle stiffness is present

T or F: During weight bearing movements from extension into flexion the femur and tibia are supported primarily by the anterior menisci

False During weight bearing movements from extension into flexion the femur and tibia are supported primarily by the *posterior* menisci

T or F: wand exercises are the only types of exercise that should be used to regain assisted motion

False - *wand exercises or pulley system exercises*

Acetabular labral tears are often associated with what (especially when the knee is flexed and there is hip rotation)?

Femoroacetabular impingement syndrome.

What tendons are involved with medial humeral epicondylitis?

Flexor carpi radial is, pronator teres, and flexor carpi ulnaris tendons.

Tendons attaching at medial epicondylitis

Flexor carpi radialis Pronator Teres Flexor carpi ulnaris

Jersey Finger aka

Flexor digitorum profundis avulsion

How do lateral and medial humeral epicondylitis occur?

Frequently in daily activities as a result of the repetitive loads encountered and in athletes from both repetitive and forceful muscular activations inherent in throwing, hitting, serving, and spiking.

Aka for Adhesive capsulitis

Frozen Shoulder

Ulnar Collateral Ligament rupture is also called

Game Keepers Thumb OR Skier's Thumb

Gluteal tears most commonly affect people in their ______ decades of life and ________ 4x more than ________.

Gluteal tears most commonly affect people in their *4th-6th* decades of life and *women* 4x more than *men*.

Groin injuries are hard to diagnose, so ________ and ________ are important.

Groin injuries are hard to diagnose, so *location* and *mechanism of injury* are important.

Groin injuries are usually due to an ________, ________ injury and usually have ________ symptoms that are non-musculoskeletal in origin.

Groin injuries are usually due to an *acute, musculoskeletal* injury and usually have *chronic* symptoms that are non-musculoskeletal in origin.

Longitudinal tears have a more favorable ________ compared to radial tears.

Healing potential.

Around day 5-7, change from a splint to a _______________ from 30-90 degrees & __________ ROM begins.

Hinged elbow brace Active (Requires muscle activation & increased stability & compression)

What are clinical presentations for an elbow dislocation?

History of a motor vehicle accident, direct trauma, sports injuries, and other high-energy mechanisms account for a minority of dislocations in young individuals. Soft tissue swelling and obvious deformity. The constellation of elbow dislocation with concurrent fractures of the radial head and the coronoid process has been termed "the terrible triad," suggesting the poor outcomes associated with its treatment.

How do you evaluate for an MCL injury of the elbow?

History of chronic elbow pain with throwing during late cocking phase and early acceleration. Possibly can't throw anymore due to pain. Valgus laxity of the elbow. MRI will show damage to MCL Arthroscopic test will show 1mm or greater of space between the coronoid and the medial humerus

What are clinical presentations of post traumatic elbow stiffness.

History of traumatic events, previous surgery, and septic arthritis. Comorbid conditions such as hemophilia, which causes hemarthroses, or a spastic neuropathy which may result in neuropathic joint degeneration. Heterotropic Ossification- direct trauma, neural axis injury, surgical intervention, and forceful passive manipulation may cause HO. Attention to a soft or hard end point at the extremes of each motion is paramount to determining whether a soft tissue or bony constraint is hindering motion. If there is crepitus through range of motion it may indicate loose body, fracture, or degenerative changes.

If conservative treatment for trochanteric bursitis is unsuccessful then you need to rule out ________ with ________.

If conservative treatment for trochanteric bursitis is unsuccessful then you need to rule out *gluteal tear* with *MRI*.

If extensor lag is present, you know that the ________ or ________ are not likely contributors to the injury.

If extensor lag is present, you know that the *joint* or *muscle stiffness* are not likely contributors to the injury.

Why does extensor lag occur?

If joint or muscle stiffness is present then passive ROM will be affected. It may occur because of edema, pain, or weakness.

If someone does not have a PCL, they will have less engagement of the ________ muscle and may also have a positive ________ orthopedic test.

If someone does not have a PCL, they will have less engagement of the *quadriceps* muscle and may also have a positive *anterior drawer* orthopedic test.

Injuries to the femoral origin of the MCL can be seen with an increase in what?

Increase in the normal prominence of the medial epicondyle.

With adductor strain, acute tears at the musculotendinous junction can stand relatively ________ rehabilitation. Partial tears at the tendinous insertion usually requires a period of ________ before ________ treatment can occur.

With adductor strain, acute tears at the musculotendinous junction can stand relatively *aggressive* rehabilitation. Partial tears at the tendinous insertion usually requires a period of *rest* before *pain-free* treatment can occur.

Scapulothoracic dyskinesia affects the ______________ shoulder capsule.

Inferior/posterior

Tendinitis sx

Inflammation SHARP

What are syndesmotic injuries?

Injuries involving disruption of the ligamentous structures between the distal fibula and tibia, just proximal to the ankle joint. -May occur as a purely soft tissue injury or in association with an ankle fracture.

A large effusion is suggestive of an ________ injury.

Intra-articular

Post-Traumatic Elbow Stiffness intrinsic causes

Intra-articular pathology from 1. Deformities or malalignment of articular surface 2.Intra-articular adhesions 3. Loose bodies 4. Impinging osteophytes 5. Fibrosis within olecranon or coronoid fossa

Describe 2 causes of post traumatic elbow stiffness.

Intrinsic causes. Related to intra-articular pathology resulting from deformities or malalignment of the articular surface, intra-articular adhesions, loose bodies, impinging osteophytes, and fibrosis within the olecranon or coronoid fossa. Extrinsic causes. Related to all entities aside from the articular surface such as skin contracture from scars or burns, capsular and collateral ligament contracture, and heterotopic ossification. Another important ext inside cause is injury to brachial is or triceps resulting in a hemarthrosis, which may cause scarring, fibrosis, and limitation of motion.

Sprains to both the ATFL and CFL are a result of combined ________ and ________.

Inversion and Plantarflexion.

Basic safe exercises to begin mm strengthening includes ______________ & ________________.

Isometrics Closed kinetic chain exercises

Strengthening of the shoulder can progress by having the patient do more aggressive exercises which include _______________ & _______________.

Isotonic Open chain exercises

What is the Warren and Marshall three-layer concept?

It describes the anatomic structures of the medial side of the knee. -The first layer is composed of the fascia investing the sartorius muscle. -The second layer contains the superficial MCL, the medial patellofemoral ligament, and the ligaments of the posteromedial corner of the knee. -The third layer is formed from the true capsule of the knee joint and the deep MCL, which is composed of the meniscofemoral and meniscotibial ligaments.

How does excessive rearfoot pronation influence patellar alignment?

It increases tibial medial rotation and changes the quadriceps tendon pull on the patella.

What is a bucket handle tear?

It is a severe form of vertical tear where the central part of cartilage tissue gets detached from the tibia bone and is displaced from its position. Thus, a flap is formed that looks very similar to a bucket handle.

Is weightbearing allowed in phase 1 MCL rehab?

It is allowed as tolerated.

It is proposed that ________ kinetic chain exercise is safer for grafted symptoms for ACL reconstruction due to the low ________ sheer placed on the knee. Ex//Co-contraction of ________ and ________ with closed chain.

It is proposed that *closed* kinetic chain exercise is safer for grafted symptoms for ACL reconstruction due to the low *anterior* sheer placed on the knee. Ex//Co-contraction of *quadriceps* and *hamstring* with closed chain.

What is the Pivot Shift test?

It tries to mimic the giving away mechanism. We learned it by its aka: Macintosh test.

Describe the condition: avulsion of the flexor digitorum profundus

Jersey Finger

Johnston et al. (1998) suggested that most patients with internal snapping hip syndrome present with hip ________ tightness and hip ________ weakness.

Johnston et al. (1998) suggested that most patients with internal snapping hip syndrome present with hip *flexor* tightness and hip *rotator* weakness.

AMI stays longer and is more severe with ________, so the number one goal with first stage rehabilitation efforts is to reduce it.

Joint effusion

What is conservative treatment for an MCL/ulnar nerve injury?

Joint mobilization and low-load, long-duration stretching are advocated for restoring of extension. High-intensity, short-duration stretching is contraindicated for limited elbow ROM (may produce heterotopic ossification) Initial treatment includes moist heat and ultrasound, dynamic splinting at night during sleep (low-load, long-duration stretch), joint mobilization's, and ROM exercises at end ranges done several times a day

Changes at the hip or at the foot cause compensatory changes and increase stress at the ________.

Knee

Treatment for Lateral/Medial humeral epicondylitis includes _____________ & ____________.

LLLT (low level laser therapy) Plyometrics

Labral lesions lead to ________ of the joint and early ________ changes.

Labral lesions lead to *instability* of the joint and early *arthritic* changes.

What is the best orthopedic test (gold standard) to determine ACL integrity and see anterior translation? Why?

Lachman's test because the 20-30 degree angle will relax secondary stabilizers.

The MCL helps to restrict ________ rotation of the tibia on the femur.

Lateral

_____________ epicondylitis is most common.

Lateral

The squeeze test and external rotation test are useful for diagnosis of purely ________ injuries.

Ligamentous

What is the most common complication with elbow dislocations?

Loss of terminal extension with contractures in up to 60% of cases

What are conservative treatments for epicondylitis?

Low-level laser (LLLT) offer short-term pain relief and less disability in humeral epicondylitis, both alone and in conjunction with an exercise. LLLT and plyometrics were effective treatment.

Scapular strengthening should include ______________ & ________________.

Lower trap Serratus anterior

Which ligament of the elbow is the primary restraint to valgus stress?

MCL

Gold standard for imaging diagnosis for a MCL injury?

MRI

Describe the condition: avulsion of the extensor tendon from its distal insertion

Mallet finger

The inability to actively extend or straighten the DIP joint is associated to what condition?

Mallet finger

What are provocative test for lateral and medial epicondylitis?

Manual muscle test to determine pain reproduction. Wrist and finger flexion and extension and forearm pronation and supination. Testing of the elbow at or near full extension can often recreate localized lateral or medial elbow elbow pain secondary to tendon degeneration. Reproduction of lateral or medial elbow pain with resistive muscle testing may indicate associated tendon injury at the elbow and would direct the clinician to perform s more complete elbow examination.

Meniscal posterior horn and lateral tears are primarily ________ tears, whereas anterior horn tears are often ________ tears.

Meniscal posterior horn and lateral tears are primarily *degenerative* tears, whereas anterior horn tears are often *traumatic* tears.

Who tends to get Pincer impingement?

Middle aged females.

Carpal Tunnel Syndrome occurs most commonly during __________

Middle and advanced age

How common are elbow dislocations in children and adults?

Most common dislocation in children younger than 10 years old. 2nd most common elbow dislocation in adults.

What is the flexion contracture in throwing athletes?

Most often a result of valgus extension overload syndrome. Repetitive near-tensile failure loads sustained by the anterior bundle of the ulnar collateral ligament in late cocking/early acceleration result in attenuation or rupture and subsequent valgus instability. Increased contact stress between the radial head and capitellum in addition to the medial ole random fossa and olecranon. Reactive osteophytes develop on the proximal olecranon and corresponding olecranon fossa (kissing osteophytes), which subsequently impinge and limit terminal extension. Occasionally, hypertrophic osteophytes may fracture and form loose bodies, further limiting extension.

What orthopedic test will be positive with external coxa saltans (IT band syndrome)?

Ober's

Flexion Contracture is common in repetitive _____________ loads sustained by _______________ of UCL in late __________& early acceleration phase, which will cause the UCL bundle to ___________ or __________, & valgus instability.

Near-tensile failure Anterior bundle Cocking Attenuate Rupture

________ muscle inhibition prevents the quad from firing properly with AMI, therefore causing a delay in rehabilitation.

Neurogenic

Sx of Subtle GH instability

Non-descript level of discomfort & diffuse pain @ shoulder girdle Poorly localized & may be scapular & at posterior joint line or anterior subacromial (mimic rotator cuff discomfort) Arm overhead leads to PTN down arm w/o Dermatomal pattern Hx of repetitive microtrauma

What is femoroacetabular impingement?

Occurs when the acetabulum and the femur contact each other prematurely during hip motion. -Can involve Pincer or Cam impingement. -Occurs in the anterior and superior aspect of the hip.

How are intra-articular pathologies at the hip usually treated?

Often requires surgical management because there is a decrease of blood supply in the hip which will reduce the success of conservative management.

If conservative treatment is not working for chronic ankle instability then what is the next step?

Operative treatment to reduce OA

___________________ provide specific sensory inputs that facilitate activity or movement pattern.

PNF exercises

What are the clinical signs and symptoms of tarsal tunnel syndrome?

Pain and numbness or tingling in medial ankle radiating into plantar aspect of foot only. No dorsal foot numbness or tingling (consider peripheral neuropathy if dorsal numbness present).

What clinical findings are associated with syndesmotic injury?

Pain anteriorly between the distal tibia and fibular and posteromedially at the level of the ankle joint. The pain is worse when bearing weight or pushing off the ground.

What is the typical patient complaint with meniscal injury?

Pain inside knee or along the joint line "catches or locks."

Manual muscle tests determine ______________ - do _______________ flexion/extension, and _____________ pronation/supination.

Pain reproduction Wrist Forearm

People with PFPS are likely to develop ________ later in life.

Patellofemoral osteoarthritis.

1st sign of frozen shoulder is the ______________________, followed by ____________ discomfort.

Patient can't reach behind back (internal rotation) Posterior shoulder

A part of the following statement is incorrect. Find the right word(s) to make the sentence correct: Patient's with a LCL injury may report a popping or tearing sensation.

Patient's with a *MCL* injury may report a popping or tearing sensation *on the medial aspect of the knee*

Which type of perturbation is easier for the patient: patient-initiated or doctor-initiated?

Patient-initiated

What condition is described: stenosis of the A1 pulley of thumb in infants causes locking in flexion

Pediatric trigger thumb

What does the following describe: because of the bone spurs on the rim of the acetabulum this will pinch the labrum between the rim and femoral head during flexion

Pincer impingement

ROM of the knee before ____ degrees of flexion has little effect on meniscal ________, but flexion angles greater than ____ degrees translate the menisci ________. This increased translation may place detrimental stresses across a healing meniscus, this is why you would perform wall sits at less than ____ degrees (mini squats).

ROM of the knee before *60* degrees of flexion has little effect on meniscal *displacement*, but flexion angles greater than *60* degrees translate the menisci *posteriorly*. This increased translation may place detrimental stresses across a healing meniscus, this is why you would perform wall sits at less than *50* degrees (mini squats).

How may the patient describe an MCL injury?

Popping or tearing sensation on the medial aspect of the knee. -An absence of an effusion may indicate a severe tear, which allows fluid to extravasate outside the joint and into the surrounding tissues.

Which orthopedic test is the most sensitive test for PCL integrity?

Posterior drawer test at 90 degrees of knee flexion.

Valgus over pressure test causing posterior elbow pain indicates __________________.

Posterior medial osteophytes

What is the level A recommendation (strong evidence) to treating plantar fasciitis?

Prefabricated or custom foot orthoses can provide short-term (3 months) reduction in pain and improvement in function.

Flexion Contracture increases contact stress between _____________ & ____________ & _________________ & the _____________.

Radial head Capitulum Medial olecranon fossa Olecranon

GH instability is the m/c pathology ______________ in patients younger than 30.

Producing symptom

What is the goal of nonsurgical management of intra-articular hip pathology?

Protect the damaged structures and reduce the acute symptoms.

Terminal extension to 0 degrees can be difficult for a weakened ________ to achieve.

Quadriceps

Flexion Contracture guidelines for a safe Return to Play: -painless and full ______ -no __________ -satisfactory __________ _________ strength testing -satisfactory clinical ____________

ROM Tenderness Isokinetic muscular examination

What is the mechanism of injury for meniscal involvement?

Rotating or twisting or degenerative.

What is RSDS?

Reflex Sympathetic Dystrophic Syndrome: Pain in arm or leg following injury or HA, stroke.

Ganglion cysts mostly originate around ______

Scapholunate interval

What is the MOI for theMCL/ulnar nerve injury

Repetitive flexion and extension with valgus stress. This injury is often seen in throwing athletes such as baseball pitchers, quarterbacks and javelin throwers. Cause by large valgus torque at the elbow (angular velocity from flex to ext recorded 3000/sec). Repetitive overloading can lead to microtears and inflammation of the ligament and may cause failure. It may also cause injury to the ulnar nerve which is exacerbated if the joint is unstable.

Lateral & Medial humeral epicondylitis is common in daily activities with ______________ or athletes with __________ & _______________ activations.

Repetitive load Repetitive Forceful

What is the mechanism of injury for femoroacetabular impingement?

Repetitive pivoting, twisting, and cutting maneuvers with repetitive hip flexion.

What is the mechanism of injury for acetabular labral tears?

Repetitive pivoting, twisting, and cutting maneuvers with repetitive hip flexion. -Ex// Gymnast, swimmer breast stroke, basketball, tennis, dancers, long jumping, hockey players/skaters.

Restoration of at least ____% strength and a ________ full ROM are criteria for return to sport. This may require ________ weeks for an acute strain and up to ________ months for a chronic injury.

Restoration of at least *70*% strength and a *pain-free* full ROM are criteria for return to sport. This may require *4-6* weeks for an acute strain and up to *6* months for a chronic injury.

What is the goal of phase 2 MCL rehab?

Restoration of the strength of the injured leg to approximately 80-90% of the uninjured leg.

Intermediate & advanced phases of treatment for MCL injuries will focus on ____________, strengthening, and ___________ of the entire UE.

Restoring full ROM Flexibility

Define a Grade II ankle sprain.

Result in a partial tearing of the ligamentous fibers and are considered to be moderate sprains.

Define a Grade III ankle sprain.

Result in substantial tearing of the ligamentous fibers and are considered severe sprains.

Define a Grade I ankle sprain.

Results in a stretching of the ligamentous fibers and are considered minor sprains.

Make the following statement correct: Return to normal play is allowed once a patient is able to stand on their involved leg without issue.

Return to normal play is allowed *once functional testing and functional sport specific training* is performed

Jersey Finger is most common in what finger

Ring finger

Scapular strengthening can be done with ______________ & ________________.

Seated/standing rows plus position closed chain (scap protraction)

The concave tibia moves on the convex femur in the ________ direction as the physiologic movement of the joint.

Same

LLLT has been shown to decrease _______________ & _____________, both alone & with plyometric exercises.

Short term pain Disability

What are rehabilitation exercises for humeral epicondylitis?

Slow progressive eccentric scapular strengthening exercises are recommended that target strengthening of the lower trapezius and serrated anterior force couple. Seated/standing rows. Emphasized external rotation with retraction, an exercise shown to recruit the lower trapezius at a rate 3.3 times more than the upper trapezius and utilize the important position of scapular retraction. Plus-Position closed chain exercises. Maximal scapular protraction, has been recommended for its inherent maximal serrated anterior recruitment. Strengthen posterior rotator cuff to provide strength, fatigue resistance, and optimal muscle balance. Prone horizontal abduction exercise- supraspinatus mm. Use cuff weight and attached proximal to elbow if the distal weight attachment provokes pain or stresses the healing elbow structures.

Aka for Coxa Saltans

Snapping Hip

When the UCL ligament is displaced and has a palpable mass - this is called?

Stener lesion

Trigger finger aka

Stenosing flexor tenosynovitis

MCL injuries tend to become _________ which could be an inflammatory response of the ___________ to trauma, fibrosis of the _________________, and the joint is traversed by ____________ rather than ___________.

Stiff Anterior capsule Flexor/pronator group Muscles Tendons

After 6-8 weeks following the elbow dislocation, you can begin ______________.

Strengthening

Strengthen posterior rotator cuff with _________, _________ and ____________.

Strengthening Fatigue resistance Optimal mm balance

Subsequent ACL injuries demonstrated increased ________ forces and high knee abduction loads when ________ from a jump.

Subsequent ACL injuries demonstrated increased *dynamic knee valgus* forces and high knee abduction loads when *landing* from a jump.

Prone horizontal abduction strengthens _______________ & with elbow dysfunction, use cuff weight at the ____________ if holding it ____________ increases pain or stresses the elbow.

Supraspinatus Proximal elbow Distal

T or F: PCL tear is best detected at 70 - 90 degrees of knee flexion

TRUE

Bracing and ________ may have some stabilizing effects on chronic ankle instability.

Taping

What is a SLAP lesion?

Tear of the glenoid labrum

Tendinitis or Tendinosis/Tendinopathy? Inflammatory

Tendinitis

Tendinitis or Tendinosis/Tendinopathy? Presence of SHARP

Tendinitis

Describe the differences between tendinitis and tendinitis.

Tendinitis is inflammation with the presence of sharp. Tendinitis/tendinopathy is chronic degeneration, regeneration, and microtears of the tendinous tissue

Tendinitis or Tendinosis/Tendinopathy? May be painful

Tendinosis/Tendinopathy

Tendinitis or Tendinosis/Tendinopathy? chronic degeneration, regeneration, and microtears of the tendinous tissue

Tendinosis/Tendinopathy

Recreation at or near full extension local to lateral/media epicondyle, is secondary to __________________.

Tendon degeneration

Pain with resistance during mm tests indicates associated _____________.

Tendon injury

What do extra-articular pathologies of the hip tend to involve?

Tendons, muscles, bursae, fascia, and nerves. -Imaging studies or arthroscopy may be necessary to establish a final diagnosis.

What should you test on women with PFPS? What should you implement as part of the rehab regimen?

Test hip strength and implement strengthening exercises.

A part of the following statement is incorrect. Find the right word(s) to make the sentence correct: The presence of an effusion may indicate a severe tear.

The *absence* of an effusion may indicate a severe tear *which allows fluid to extravasate outside the joint and into the surrounding tissues*

The ACL protects the knee from ________ translation of the tibia on the femur. The PCL restricts knee ________ and ________ translation of the tibia on the femur.

The ACL protects the knee from *anterior* translation of the tibia on the femur. The PCL restricts knee *hyperextension* and *posterior* translation of the tibia on the femur.

The PCL functions as the primary restraint to ________ translation of the tibia and a secondary restraint to ________ rotation.

The PCL functions as the primary restraint to *posterior* translation of the tibia and a secondary restraint to *external* rotation.

T or F: Phalen maneuver can be used to diagnose carpal tunnel syndrome

True

T or F: Posterior translation and external rotation is far more severe when both PCL tear and posterolateral corner tear is present

True

T or F: Rotator cuff injuries in the elderly are often caused from a fall

True

T or F: both diabetes and hypothyroidism are significant co-morbidities for frozen shoulder

True

T or F: chronic groin injuries that are chronic are often non-musculoskeletal in origin

True

T or F: frozen shoulder progression is freezing > frozen > thawing

True

T or F: if my knees are closer together my Q-angle is larger than if they were farther apart.

True

T or F: majority of the stability at shoulder is determined by ST structures that cross it

True

The collateral ligaments help protect and stabilize against ________ and ________ stresses.

The collateral ligaments help protect and stabilize against *medial* and *lateral* stresses.

T or F: pain associated to SLAP lesions is described as located in the posterior shoulder and is deep

True

T or F: the main key to recovery is to normalize motion

True

T or F: both Femoroacetabular impingement and acetabular labral tears have similar MOI

True *repetitive pivoting, twisting and cutting maneuvers with repetitive hip flexion*

T or F: gluteal teats will present with weak hip abductors

True *+ve Trendenlenburg gait *leaning over involved LE during stand phase *passive abduction of the hip to decrease load to the gluteals

T or F: Percussion Test can be used to diagnose carpal tunnel syndrome

True *percussion = tinel*

T or F: the first signs of a frozen shoulder in a patient is being unable to perform Apley's II

True - not being able to reach behind the back Apley's II = internal rotation and adduction

Where does the medial collateral ligament (MCL) attach?

The medial meniscus.

True/false: Isolated PCL tear does not cause varus-valgus laxity or increased rotation.

True. If there is varus or valgus laxity in extension, then by definition there is combined injury to the PCL and collateral complex.

Describe the 5 types of post traumatic elbow stiffness.

Type 1 involves soft tissue contracture. Type 2 involves soft tissue contractures with ossification. Type 3 involves non displaced articular fracture with soft tissue contracture. Type 4 involves displaced intra-articular fractures with soft tissue contracture. Type 5 involves post-traumatic bony bars blocking elbow motion.

If someone has a labral tear that has gone undiagnosed for a long time, why do they finally go see a doctor?

The resultant degeneration is finally causing pain.

How does the height of a person effect Q angle?

The taller the person, the smaller the Q angle.

What does it mean if the knee hyperextends asymmetrically?

There is combined cruciate and posterolateral corner injury.

How does the vascular supply to the menisci at the knee effect management of injuries?

There is not a good blood supply, which is why degenerative changes and injuries don't heal well.

What is extensor lag?

This is where full passive motion is present but active terminal knee extension does not occur. -It is a protective mechanism of the knee. -It is mainly a diagnostic tool.

Flexion Contracture found in _________ Athletes resulting from valgus ______________ overload syndrome

Throwing Extension

With external coxa saltans (IT band syndrome), pain is experienced when an individual is lying on the ________ side and during ________ transitions. Pain increases with palpation over the ________ and with external rotation of the hip in ____ degrees of flexion.

With external coxa saltans (IT band syndrome), pain is experienced when an individual is lying on the *affected* side and during *sit-to-stand* transitions. Pain increases with palpation over the *greater trochanter* and with external rotation of the hip in *90* degrees of flexion.

Some orthopedic testing to help DDX: _______ test- ulnar nerve irritation _______ Stress test- integrity of the UCL _______ Stress test- integrity of the LCL ______ ___________ Test- post elbow pain for posteromedial osteophytes

Tinel Valgus Varus Valgus Overpressure

Positive Ortho tests for Lateral/Medial Humeral Epicondylitis

Tinel test (ulnar n) Valgus stress (MCL) Varus stress (LCL) Valgus over pressure test

Sx reflective of GH instability had a ______________.

Traumatic origin

What condition is described: commonly occurs at thumb, middle finger or ring finger

Trigger finger

What condition is described: painful snapping which occurs at the finger flexor tendons suddenly pull through a tight A1 pulley

Trigger finger

T or F: Combined cruciate and posterolateral corner injury usually occurs when the knee hyperextends asymmetrically

True

T or F: Do not stretch the ITB during the initial stage of inflammation of an external coxa saltans

True

T or F: Finkelstein maneuver is an ortho test for De Quervain Tenosynovitis

True

T or F: Hand diagrams and Electromyography are used to diagnose carpal tunnel syndrome

True

T or F: PNF exercises like D2 Flexion-Extension exercises for UE can enhance the stability of the glenohumeral joint

True

What are the clinical signs and symptoms of plantar fasciitis rupture?

Typically antecedent plantar fasciitis symptoms, with a pop or "crunch" during push-off or pivoting, then severe pain with subsequent inability to bear weight (or only with difficulty).

De Quervain Tenosynovitis occurs with repetitive forceful hand or wrist movements with _____ deviation of the wrist

Ulnar

What should you do if you have a patient that has a high static Q angle?

Work the VMO and hip abductors.

MCL caused by large ______ torque at the _______.

Valgus at the elbow (angular velocity from flex to ext recorded 3000/sec)

The m/c MOI for flexion contracture in throwing athletes is _____________________ Syndrome.

Valgus extensioin overload

A patient with MCL laxity/injury will have a positive _____________ test.

Valgus stress

What is the crucial test for evaluating an injury to the MCL?

Valgus stress test with the knee in 30 degrees of flexion.

Describe the blood supply to the menisci at the knee.

Vascular except for the lateral 25%.

What is Pincer impingement?

When the acetabulum covers more of the femoral head than it should. -Causes the anterosuperior aspect of the acetabulum to pinch the labrum between the rim and femoral head during flexion.

Triangular Fibrocartilage Complex (TFC) injury is often seen in what types of athletes

Weightlifters and Gymnasts

How has a zero-gravity treadmill affected post-surgical rehabilitation of the knee.

You can take someone 2-4 hours post knee surgery and get them walking on a zero-gravity treadmill. This is a "game-changer" for knee rehab necause it allows the old waste to leave and new nutrients to come in.

What is the step down test (SLS test)?

You have the patient step down... -Watch the angle.

Who tends to get Cam impingement?

Young male athletes. -Usually due to an impact injury to a growth plate.

Match the elbow dislocation rehab with the stage: The first 5-7 days a. elbow is splinted to allow soft tissue rest b. a hinged elbow brace from 30 to 90 degrees is applied and active ROM is initiated c. strengthening

a. elbow is splinted to allow soft tissue rest

Lateral epicondylitis tendon is which of the following? a. extensor carpi radialis brevis b. flexor carpi radialis brevis

a. extensor carpi radialis brevis (Additionally, the extensor carpi radialis longus and extensor carpi ulnaris can be involved)

Flexion Contracture initial treatment includes what? a. moist heat and ultrasound, dynamic splinting at night during sleep, joint mobilizations, and ROM exercises at end ranges done several times a day b. high-intensity, short-duration stretching c. Joint mobilization and low-load, long-duration stretching

a. moist heat and ultrasound, dynamic splinting at night during sleep, joint mobilizations, and ROM exercises at end ranges done several times a day (b. is contraindicated & c. is a conservative treatment but her slides didn't indicate that it was an initial treatment)

Post Traumatic Elbow Stiffness Match the following with the Type: soft tissue contractures a. type 1 b. type 2 c. type 3 d. type 4 e. type 5

a. type 1

The combination of elbow dislocation with concurrent fractures of the radial head and the coronoid process has been termed what? a. "the triple threat" b. "the terrible triad" c. "threat level midnight" d. "three's company"

b. "the terrible triad" (suggesting the poor outcomes associated with its treatment)

What is the prevalence percentage of an elbow dislocation injury? a. 6-18% b. 11-28% c. 15-38%

b. 11-28%

What test is done for a 1mm or greater of space between the coronoid and the medial humerus? a. X-ray b. Arthroscopy c. MRI

b. Arthroscopy

Repetitive overloading may also cause injury to which nerve? a. Radial b. Ulnar c. Median

b. Ulnar

Match the elbow dislocation rehab with the stage: Beginning at day 5-7 a. elbow is splinted to allow soft tissue rest b. a hinged elbow brace from 30 to 90 degrees is applied and active ROM is initiated c. strengthening

b. a hinged elbow brace from 30 to 90 degrees is applied and active ROM is initiated

During the intermediate and advanced phases of rehabilitation of a MCL injury the goal is to what? a. promote soft tissue healing b. restore full ROM, strength, and flexibility of the entire upper extremity c. restore valgus stress résistance to the elbow

b. restore full ROM, strength, and flexibility of the entire upper extremity

Post Traumatic Elbow Stiffness Match the following with the Type: soft tissue contractures with ossification a. type 1 b. type 2 c. type 3 d. type 4 e. type 5

b. type 2

Clinical Examination of humoral epicondylitis: Lack of _____________ symmetry in the upper extremities Decreased motion in ______ and _______ with a cubitus _________ deformity

bilateral flexion and extension valgus

Medial humeral epicondylitis primary site is which of the following? a. flexor carpi radialis b. pronator teres c. flexor carpi ulnaris tendons d. all of the above

d. all of the above

Which of the following are clinical presentations of a MCL injury? a. Hypertrophied flexor-pronator mass b. 4-5th digit numbness or tingling c. postmedial elbow pain d. all of the above

d. all of the above

What are the initial goals for conservative Treatment/Rehabilitation of a MCL injury? a. reduce pain and inflammation b. promote soft tissue healing c. avoid loss of ROM d. all of the above

d. all of the above (Acute, traumatic injuries are sometimes braced; chronic, throwing injuries are not)

Post Traumatic Elbow Stiffness Match the following with the Type: displaced intra-articular fractures with soft tissue contracture a. type 1 b. type 2 c. type 3 d. type 4 e. type 5

d. type 4

Post Traumatic Elbow Stiffness Match the following with the Type: involves post-traumatic bony bars blocking elbow motion a. type 1 b. type 2 c. type 3 d. type 4 e. type 5

e. type 5

Lateral and Medial Humeral Epicondylitis occurs ___________ as a result of ____________ loads

frequently repetitive

Post Traumatic Elbow Stiffness Comorbid condition that can lead to hemarthroses or spastic neuropathy?

hemophilia

Functional progressions will include_______ and ________ rotation exercises to avoid excessive valgus stress to the elbow

internal and external

Tx of Tendinitis or Tendinosis/Tendinopathy

low level laser therapy (LLLT)

Repetitive overloading (flexion/extension) of the MCL can lead to __________ and __________ of the ligament

micro-tears and inflammation

Medial Collateral Ligament seen in ________ athletes.

throwing athletes (most common in baseball pitchers, quarterbacks and javelin throwers)

What can cause Heterotopic Ossification? Direct ________, _______ axis injury, _________ intervention, and _______ passive manipulation

trauma neural surgical forceful (Direct trauma, neural axis injury, surgical intervention, and forceful passive manipulation may cause HO)

Clinical Presentation of history of Post Traumatic Elbow Stiffness (3 things TSS)

traumatic event previous surgery septic arthritis

Post Traumatic Elbow Stiffness non-surgical Tx: Static progressive adjustable splints such as the _____________ splint used for flexion/extension deficit __________-_____________ splints are most effective when used for patients with symptoms of less than 6 months to 1 year with little articular involvement

turnbuckle (f/e) supination-pronation

With a MCL injury the Valgus stress test would produce a ________ laxity. Valgus stress to elbow should be avoided in _______ phases

valgus early phases


Kaugnay na mga set ng pag-aaral

Rome: Republic to Empire Lesson Quiz 11-3

View Set

Unit 4 Study Guide American Imperialism

View Set

Chapter 8-12 Principles of Management

View Set

Chapter 10 quiz review for exam 3

View Set

Chapter 24 Chronic Respiratory problems Final exam

View Set

Microbiology, Ch 21, Nester's 9th

View Set

Hist 1302 Exam 1 Give Me Liberty

View Set